You are on page 1of 81

Ti liu bi dng i tuyn Vit Nam

tham d IMO 2010
















Thng 6 - 2010
Vietnamese IMO Team Training Camp 2010
2 | Trn Nam Dng 6/2010


Mc lc


1. Cc phng php v k thut chng minh 2
2. Nguyn l chung v th 42
3. Gii phng trnh hm bng cch lp phng trnh 50
4. Cc bi ton ti u v h cc tp hp 63
5. V k thi chn i tuyn Vit Nam d thi IMO 2010 69
6. Bt ng thc: Mt s v d v bi tp chn lc 80






















Vietnamese IMO Team Training Camp 2010
3 | Trn Nam Dng 6/2010

Cc phng php v k thut chng minh

Trong ton hc cng nh trong cuc sng, cn bit:
Linh hot x l tnh hung, chn la phng n ti u

Trn Nam Dng
Trng i hc KHTN Tp HCM

Cc nh l ton hc pht biu v cc tnh cht ca cc i tng ton hc v mi quan
h gia chng. V nhng khng nh ny cn c chng minh xut pht t cc tin ,
cc nh l v tnh cht c chng minh trc . V thc hin bc chng minh,
ta cn c nhng quy tc suy din chng minh l cht ch v mt ton hc.

Vi cc bi ton Olympic cng vy, yu cu chng minh mt kt qu no lun hin
din, ngay c trong nhng bi khng c cm t chng minh rng. Chng hn gii
phng trnh x
3
3x + 1 = 0 c th ta s phi chng minh tt c cc nghim ca chng
thuc on [-2, 2], gii phng trnh hm f(x
2
+ f(y)) = f
2
(x) + y c th ta s phi
chng minh f l ton nh ...

Bi vit ny ni v hai phng php v mt s k thut chng minh c bn: chng minh
phn chng, chng minh quy np, chng minh phn chng, dng mnh phn o,
phn v d nh nht, v d v phn v d, s dng nguyn l Dirichlet, nguyn l cc hn,
nguyn l bt bin, s dng t mu, m bng hai cch, sp xp th t

Cch tip cn ca chng ta l s thng qua cc v d ni v cc phng php v k
thut. y s ch c cc nhn xt, bnh lun, cc nguyn tc chung ch khng c
trnh by h thng nh mt l thuyt.

Bi vit ny u tin c vit v trnh by trong chng trnh Gp g g Ton hc
2010, c t chc vo thng 1 nm 2010, sau c b sung, honh chnh v trnh
by ti Hi ngh khoa hc Cc chuyn chuyn Ton v ng dng t chc ti Ba V,
thng 5/2010. Cui cng, chun b cho i tuyn Vit Nam thi Olympic Ton quc t,
bi vit c b sung thm cc phn v m bng hai cch, Nguyn l cc hn, Sp xp
th t v ng dng ca cc phng php v k thut chng minh trong bi ton Ti u t
hp.

1. Php chng minh phn chng

Mt s v d m u

Chng minh phn chng c th ni l mt trong nhng v kh quan trng ca ton hc.
N cho php chng ta chng minh s c th v khng c th ca mt tnh cht no , n
Vietnamese IMO Team Training Camp 2010
4 | Trn Nam Dng 6/2010

cho php chng ta bin thun thnh o, bin o thnh thun, n cho php chng ta l
lun trn nhng i tng m khng r l c tn ti hay khng. V d kinh in nht v
php chng minh phn chng thuc v Euclid vi php chng minh

nh l. Tn ti v s s nguyn t.

y, Euclid gi s ngc li rng tn ti hu hn s nguyn t p
1
, p
2
, , p
n
. ng xt
tch N = p
1
p
2
p
n
+ 1. N phi c t nht 1 c s nguyn t p. Khi , do p
1
, p
2
, , p
n
l
tt c cc s nguyn t nn tn ti i sao cho p = p
i
. Nhng khi p | 1, mu thun.

Bi tp

1. Chng minh rng tn ti v s s nguyn t dng 4k+3.
2. Chng minh rng tn ti v s s nguyn t dng 4k+1.

Phng php xung thang

Mt chng minh ni ting khc bng phng php phn chng chnh l chng minh ca
Euler cho nh l nh Fermat vi trng hp n = 4.

nh l. Phng trnh x
4
+ y
4
= z
4
(1) khng c nghim nguyn dng.

ng gi s rng phng trnh (1) c nghim nguyn dng. Khi , theo nguyn l
cc hn, tn ti nghim (x
0
, y
0
, z
0
) vi x
0
+ y
0
+ z
0
nh nht. Sau , bng cch s dng
cu trc nghim ca phng trnh Pythagore x
2
+ y
2
= z
2
, ng i n s tn ti ca mt
nghim (x
1
, y
1
, z
1
) c x
1
+ y
1
+ z
1
< x
0
+ y
0
+ z
0
. Mu thun.

Phng php ny thng c gi l phng php xung thang.

Bi tp

3. Chng minh rng phng trnh x
3
+ 3y
3
= 9z
3
khng c nghim nguyn dng.
4. Chng minh rng phng trnh x
2
+ y
2
+ z
2
= 2xyz khng c nghim nguyn dng.

S dng mnh phn o

Chng minh s dng mnh phn o cng l mt phng n chng minh phn chng
hay c s dng. C s ca phng php l chng minh A B, ta c th chng
minh A B . V mt bn cht th hai php suy din ny c v ging nhau, nhng trong
thc t th li kh khc nhau. Ta th xem xt 1 vi v d.

V d 1. Chng minh rng hm s
1
) (
2
+
=
x
x
x f l mt n nh t R vo R.
Vietnamese IMO Team Training Camp 2010
5 | Trn Nam Dng 6/2010

V d 2. Chng minh rng nu (p-1)! + 1 l s nguyn t th p l s nguyn t.

Trong v d 1, r rng vic chng minh x
1
x
2
suy ra f(x
1
) f(x
2
) kh khn hn vic
chng minh f(x
1
) = f(x
2
) suy ra x
1
= x
2
, d rng v mt logic, hai iu ny l tng
ng.

Trong v d 2, gn nh khng c cch no khc ngoi cch chng minh nu p l hp s, p
= r.s th (p-1)! + 1 khng chia ht cho p.

Bi tp.

5. Cho hm s f: R R tho mn cc iu kin sau
1) f n iu ;
2) f(x+y) = f(x) + f(y) vi mi x, y thuc R.
6. Cho a, b, c l cc s thc khng m tho mn iu kin a
2
+ b
2
+ c
2
+ abc = 4. Chng minh rng a + b
+ c 3.

Phng php phn v d nh nht

Trong vic chng minh mt s tnh cht bng phng php phn chng, ta c th c
thm mt s thng tin b sung quan trng nu s dng phn v d nh nht. tng l
chng minh mt tnh cht A cho mt cu hnh P, ta xt mt c trng f(P) ca P l
mt hm c gi tr nguyn dng. By gi gi s tn ti mt cu hnh P khng c tnh
cht A, khi s tn ti mt cu hnh P
0
khng c tnh cht A vi f(P
0
) nh nht. Ta s
tm cch suy ra iu mu thun. Lc ny, ngoi vic chng ta c cu hnh P
0
khng c
tnh cht A, ta cn c mi cu hnh P vi f(P) < f(P
0
) u c tnh cht A.

V d 3. Cho ng gic li ABCDE trn mt phng to c to cc nh u nguyn.
a) Chng minh rng tn ti t nht 1 im nm trong hoc nm trn cnh ca ng gic
(khc vi A, B, C, D, E) c to nguyn.
b) Chng minh rng tn ti t nht 1 im nm trong ng gic c to nguyn.
c) Cc ng cho ca ng gic li ct nhau to ra mt ng gic li nh A
1
B
1
C
1
D
1
E
1

bn trong. Chng minh rng tn ti t nht 1 im nm trong hoc trn bin ng gic li
A
1
B
1
C
1
D
1
E
1
.

Cu a) c th gii quyt d dng nh nguyn l Dirichlet: V c 5 im nn tn ti t nht
2 im X, Y m cp to (x, y) ca chng c cng tnh chn l (ta ch c 4 trng hp
(chn, chn), (chn, l), (l, chn) v (l, l)). Trung im Z ca XY chnh l im cn
tm.

Sang cu b) l lun trn y cha , v nu XY khng phi l ng cho m l cnh th
Z c th s nm trn bin. Ta x l tnh hung ny nh sau. rng nu XY l mt
cnh, chng hn l cnh AB th ZBCDE cng l mt ng gic li c cc nh c to
u nguyn v ta c th lp li l lun nu trn i vi ng gic ZBCDE, Ta c th
Vietnamese IMO Team Training Camp 2010
6 | Trn Nam Dng 6/2010

dng n bin chng minh qu trnh ny khng th ko di mi, v n mt lc no
s c 1 ng gic c im nguyn nm trong.

Tuy nhin, ta c th trnh by li l lun ny mt cch gn gng nh sau: Gi s tn ti
mt ng gic nguyn m bn trong khng cha mt im nguyn no (phn v d). Trong
tt c cc ng gic nh vy, chn ng gic ABCDE c din tch nh nht (phn v d nh
nht). Nu c nhiu ng gic nh vy th ta chn mt trong s chng. Theo l lun
trnh by cu a), tn ti hai nh X, Y c cp to cng tnh chn l. Trung im Z
ca XY s c to nguyn. V bn trong ng gic ABCDE khng c im nguyn no
nn XY phi l mt cnh no . Khng mt tnh tng qut, gi s l AB. Khi ng
gic ZBCDE c to cc nh u nguyn v c din tch nh hn din tch ng gic
ABCDE. Do tnh nh nht ca ABCDE (phn v d nh nht pht huy tc dng!) nn bn
trong ng gic ZBCDE c 1 im nguyn T. iu ny mu thun v T cng nm trong
ng gic ABCDE.

Bi tp

7. Gii phn c) ca v d 3.
8. (nh l Bezout) Chng minh rng nu (a, b) = 1 th tn ti u, v sao cho au + bv = 1.
9. Trn mt phng nh du mt s im. Bit rng 4 im bt k trong chng l nh ca mt t gic li.
Chng minh rng tt c cc im c nh du l nh ca mt a gic li.

Phn chng trong cc bi ton chng minh s khng tn ti

Phng php phn chng thng hay c s dng trong cc bi ton bt bin hoc bi
ton ph hnh chng minh s khng thc hin c. Sau y chng ta xem xt 2 v d
nh vy.

V d 4. Xt hnh vung 7 7 . Chng minh rng ta c th xo i mt phn cn li
khng th ph kn bng 15 qun trimino kch thc 1 3 v 1 qun trimino hnh ch L.

Ta chng minh rng nu b i mt gc trn bn tri th phn cn li khng th ph
c bng cc qun trimin cho.

lm iu ny, ta nh s cc vung nh sau

1 2 3 1 2 3 1
1 2 3 1 2 3 1
1 2 3 1 2 3 1
1 2 3 1 2 3 1
1 2 3 1 2 3 1
1 2 3 1 2 3 1
1 2 3 1 2 3 1
Vietnamese IMO Team Training Camp 2010
7 | Trn Nam Dng 6/2010

Khi , nhn xt rng 1 qun trimin kch thc 1 3 s che 3 s 1, 2, 3 (nu n nm
ngang) hoc 3 s ging nhau (nu n nm dc). Nh vy tng cc s m mt qun
trimin 1 3 che lun chia ht cho 3. Trong khi d thy qun trimin hnh ch L che
3 s c tng khng chia ht cho 3.
By gi gi s ngc li rng hnh vung 7 7 b i gc trn bn tri c th ph
c bng 15 qun trimin 1 3 v 1 qun trimin hnh ch L th theo l lun trn, tng
s cc s m cc qun trimin ny che s khng chia ht cho 3. iu ny mu thun v
tng cc s trn cc cn li bng
20 1 + 14 2 + 14 3 = 90
chia ht cho 3!

Mu thun trn chng t iu gi s l sai v ta c iu phi chng minh.

V d 5. Hnh trn c bi 5 ng knh thnh thnh 10 bng nhau. Ban u trong
mi c 1 vin bi. Mi ln thc hin, cho php chn 2 vin bi bt k v di chuyn chng
sang bn cnh, 1 vin theo chiu kim ng h v 1 vin ngc chiu kim ng h. Hi
sau mt s hu hn ln thc hin, ta c th chuyn tt c cc vin bi v cng 1 c
khng?

Nu lm th th chng ta s thy rng khng th thc hin c yu cu. Chng ta c th
cng lm l dn 9 vin bi v 1 , cn 1 vin bi khc th khng th dn c. Nhng lm
th no chng minh iu ny? Li gii ha ra l kh n gin. Ta s dng phn chng
kt hp vi bt bin.

Ta t mu cc bng hai mu en trng xen k nhau. Gi S l tng s vin bi nm cc
en th trng thi ban u ta c S = 5. Nu gi s ngc li rng ta c th a cc
vin bi v cng 1 th trng thi cui cng ny, ta s c S = 0 (nu ta dn cc vin bi
v mt trng) hoc S = 10 (nu ta dn cc vin bi v mt en).
By gi ta s thu c iu mu thun nu ta chng minh c qua cc ln thc hin th
tnh chn l ca S s khng thay i, tc l nu ban u S l s l th qua cc ln thc
hin, S s lun l s l (v s khng th bng 0 hoc bng 10).

Nu nhn xt rng cc en trng xen k nhau th iu m chng ta cn chng minh kh
hin nhin v chng ti xin dnh php chng minh chi tit cho bn c.

Bi tp

10. Hnh vung 5 x 5 b i gc trn bn tri. Chng minh rng c th ph phn cn li bng 8 qun
trimino hnh ch L nhng khng th ph c bng 8 qun trimino hnh ch kch thc 1 x 3. Tm tt c
cc gi tr k sao cho c th ph phn cn li bng k qun trimino 1 x 3 v 8-k trimino hnh ch L.

11. Xt hnh vung 7 7 . Tm tt c cc m nu ta xa i th phn cn li c th ph kn bng 15
qun trimino kch thc 1 3 v 1 qun trimino hnh ch L.
Vietnamese IMO Team Training Camp 2010
8 | Trn Nam Dng 6/2010

12. Trn vng trn ban u theo mt th t tu c 4 s 1 v 5 s 0. khong gia hai ch s ging
nhau ta vit s 1 v khong gia hai ch s khc nhau ta vit s 0. Cc s ban u b xo i. Hi sau
mt s ln thc hin nh vy ta c th thu c mt b gm 9 s 0?

13. Cho trc cc hm s f
1
(x) = x
2
+ 2x, f
2
(x) = x + 1/x, f
3
(x) = x
2
- 2x . Cho php thc hin cc php
ton cng hai hm s, nhn hai hm s, nhn mt hm s vi mt hng s tu . Cc php ton ny c th
tip tc c thc hin nhiu ln trn f
i
v trn cc kt qu thu c. Chng minh rng c th thu c
hm s 1/x t cc hm s f
1
, f
2
, f
3
bng cc s dng cc php ton trn nhng iu ny khng th thc
hin c nu thiu mt trong 3 hm f
1
, f
2
, f
3
.

Phn chng trong cc bi ton bt ng thc

Trong chng minh bt ng thc, phng php phn chng thng dng o iu
kin v kt lun vi nhau trong trng hp iu kin th phc tp, cn bt ng thc cn
chng minh th n gin.

V d 1. Chng minh rng nu a, b, c l cc s thc khng m tha mn iu kin a
2
+ b
2

+ c
2
+ abc = 4 th a + b + c 3.

V d 2. (IMO 2001) Cho a, b, c l cc s thc dng. Chng minh rng ta c bt ng
thc
1
8 8 8
2 2 2

+
+
+
+
+ ab c
c
ca b
b
bc a
a


ph cc cn thc, ta t:
.
8
,
8
,
8
2 2 2
ab c
c
z
ca b
b
y
bc a
a
x
+
=
+
=
+
=
R rng x, y, z (0, 1). Ta cn chng minh rng x + y + z 1. Ch rng

2
2
2
2
2
2
2
2 2
2
2 2
2
2 2
1
.
1
.
1 512
1
1 8
,
1 8
,
1 8 z
z
y
y
x
x
z
z
ab
c
y
y
ca
b
x
x
bc
a

=

=
Nh vy, ta cn chng minh rng
x + y + z 1, trong x, y, z (0, 1) v (1-x
2
)(1-y
2
)(1-z
2
) = 512x
2
y
2
z
2

Nhng nu x + y + z < 1 th theo bt ng thc AM-GM ta c
(1-x
2
)(1-y
2
)(1-z
2
) > ((x+y+z)
2
-x
2
)((x+y+z)
2
-y
2
)((x+y+z)
2
-z
2
)
= (y+z)(y+z+2x)(z+x)(z+x+2y)(x+y)(x+y+2z)
2(yz)
1/2
.4(yzx
2
)
1/4
.2(zx)
1/2
.4(zxy
2
)
1/4
.2(xy)
1/2
.4(xyz
2
)
1/4
= 512x
2
y
2
z
2
.
Mu thun.

V d 3. Cho a, b, c, d l cc s thc khng m c tng bng 4. t
F
k
= (1+a
k
)(1+b
k
)(1+c
k
)(1+d
k
)
Chng minh rng F
4
F
3
.

Vietnamese IMO Team Training Camp 2010
9 | Trn Nam Dng 6/2010

Gi s ngc li, tn ti b bn s (a, b, c, d) tha mn: a, b, c, d 0, a + b + c + d = 4 v
F
4
< F
3
(1).

Theo bt ng thc Cauchy-Schwarz, ta c F
4
F
2
F
3
2
, F
3
F
1
F
2
2
, F
2
F
0
F
1
2
(2). T (1)
v (2) suy ra F
4
< F
3
< F
2
< F
1
< F
0
= 16 (3). T (3) ta c F
4
< 16, suy ra max(a,b,c,d) <
2.

dn ti mu thun vi (3), ta s chng minh F
3
F
1
(4). Phn ny chng minh bng
dn bin v c xem nh mt bi tp.

V d 4. (Cezar Lupu) Cho a, b, c l cc s thc dng tha mn iu kin a + b + c +
abc = 4. Chng minh rng
) .(
2
2
c b a
b a
c
a c
b
c b
a
+ +
+
+
+
+
+

Gii.
Theo bt ng thc Cauchy Schwarz, ta c
( )
2
) ( c b a
b a
c
a c
b
c b
a
b a c a c b c b a + +
|
|

\
|
+
+
+
+
+
+ + + + +
Tip tc p dng bt ng thc Cauchy Schwarz
b a c a c b c b a b a c a c b c b a c b a + + + + + + + + + + + + )) ( ) ( ) ( )( (
T suy ra

ca bc ab
c b a
c b a
b a
c
a c
b
c b
a
+ +
+ +
+ +
+
+
+
+
+
) .(
2
2

Nh vy ta ch cn cn chng minh a + b + c ab + bc + ca.
Bt ng thc Schur vi r = 1 c th vit di dng

2
) ( ) ( 4
9
c b a ca bc ab
c b a
abc
+ + + +
+ +

By gi gi s ngc li, ta c a + b + c < ab + bc + ca th
) ( )) ( 4 )( ( ) ( ) ( 4
9
2
c b a abc c b a c b a c b a ca bc ab
c b a
abc
+ + = + + + + > + + + +
+ +

Suy ra a + b + c < 3. Nhng khi abc < 1 v suy ra 4 = a + b + c + abc < 4, mu thun.

Bi tp

14. (MOP) Cho n 2 c nh. Cho x
1
, , x
n
l cc s dng tha mn iu kin

n
n
x x x
x x x
1
...
1 1
...
2 1
2 1
+ + + = + + +
Chng minh rng . 1
1
1
...
1
1
1
1
2 1

+
+ +
+
+
+
n
x n x n x n


Vietnamese IMO Team Training Camp 2010
10 | Trn Nam Dng 6/2010

15. (Pu-Ro Loh) Cho a, b, c > 1 tha mn iu kin 1
1
1
1
1
1
1
2 2 2
=

+
c b a
. Chng minh rng
. 1
1
1
1
1
1
1

+
+
+
+
+ c b a


16. Cho a, b, c l cc s thc dng tha mn iu kin c b a
c b a
+ + + +
1 1 1
. Chng minh rng
a + b + c 3abc.

17. (IMO 1991) Cho tam gic ABC v im P nm trong tam gic. Chng minh rng mt trong
cc gc PAB, PBC, PCA nh hn hoc bng 30
0
.

Mt s nh l v tnh cht chng minh bng phng php phn chng

Cui cng, ta s dng phng php phn chng chng minh mt s tnh cht quan
trng trong chng trnh ton Olympic.

nh l.
a) Nu p l s nguyn t dng 4k+1 th tn ti x sao cho x
2
+ 1 chia ht cho p;
b) Nu p l s nguyn t dng 4k+3 th khng tn ti x sao cho x
2
+ 1 chia ht cho p.
c) Nu p l s nguyn t dng 6k+1 th tn ti x sao cho x
2
+ 3 chia ht cho p;
d) Nu p l s nguyn t dng 6k+5 th khng tn ti x sao cho x
2
+ 3 chia ht cho p.

Chng minh
a) Gi s ngc li, khng tn ti x sao cho x
2
+ 1 chia ht cho p. Xt a bt k thuc
A = {1, 2, , p-1}. D dng chng minh c rng tn ti duy nht m(a) thuc A
sao cho a.m(a) -1 (mod p). Hn na, nu a b th m(a) m(b). Cui cng, do
khng tn ti x x
2
+ 1 chia ht cho p nn a m(a). Nh vy cc s 1, 2, , p-1
c phn thnh (p-1)/2 cp (a, b) vi a.b -1 (mod p). Nhn cc ng d thc
ny li vi nhau, ch k (p-1)/2 = 2k, ta c
(p-1)! (-1)
2k
1 (mod p)
iu ny mu thun vi nh l Wilson: (p-1)! -1 (mod p)!

b) Gi s tn ti x sao cho x
2
+ 1 0 (mod p)
x
2
-1 (mod p)
(x
2
)
2k+1
-1 (mod p) x
4k+2
-1 (mod p)
Mt khc, theo nh l nh Fermat, ta c
x
4k+2
1 (mod p)
T y suy ra 2 0 (mod p), mu thun. Vy iu gi s l sai, tc l khng tn ti x
sap cho x
2
+ 1 chia ht cho p.

Vietnamese IMO Team Training Camp 2010
11 | Trn Nam Dng 6/2010

c) d) c chng minh tng t da vo dy mnh tng ng sau
Phng trnh ng d x
2
+ 3 0 (mod p) c nghim
Phng trnh x
2
+ x + 1 0 (mod p) c nghim
Phng trnh x
3
1 (mod p) c nghim x 1 (mod p).

nh l.
Nu f: R R l mt hm cng tnh nhng khng tuyn tnh, th th G(f) = (x,
f(x)) tr mt trong R
2
.
C ngha l nu f(x+y) = f(x) + f(y) vi mi x, y thuc R v khng tn ti a thuc R
sao cho f(x) = ax th G(f) tr mt trong R
2
.

Chng minh. Gi s f l mt hm cng tnh nhng khng tuyn tnh. Ta t c = f(1) v
chn s thc sao cho f() c. Ta xt hm s g mi

c f
cx x f
x g

=
) (
) (
) ( .
Tnh cng tnh ca f suy ra g cng cng tnh trn R. Hn na g(1) = 0. S dng tnh cng
tnh ca g, ta suy ra rng g(q) = qg(1) vi mi q hu t. Nh vy ta c g(q) = 0 vi mi q
hu t.

Xt mt a D
r
(x, y) bt k. Chn s hu t q sao cho |q-y| < r/2 v s hu t p sao cho |p
(x-q)| < r/2. Khi ta c
(p + q -x)
2
+ (q-y)
2
< r
2
/4 + r
2
/4 = r
2
/2 < r
2
.
Nh vy im (p + q, q) nm trong a D
r
(x, y). Hn na, theo tnh cng tnh ca g, ta
c
g(p+q) = g(p) + qg() = qg() = q
Suy ra im (p + q, q) nm trn G(g), th ca g.

iu ny chng t rng mi a m trong R
2
u cha mt im no ca g. Ta v nh
vy G(g) l tr mt trong R
2
. Ta quay tr li vi f v s s dng thng tin ny. Ta c
f(x) = ug(x) + cx,
trong u = f() - c. Xt a D
r
(a, b) bt k trong R
2
. Xt a D c cho bi
D = D
s
(a, (b-c)/u),
vi } 2 1 , 2 max{ ,
2
2 2
2
c u
r
s + = =

.
V G(g) tr mt trong R
2
, ta tm c s thc y sao cho (y, g(y)) thuc D. By gi xt
im (y, ug(y) + cy) thuc G(f), php kim tra trc tip cho thy im ny thuc D
r
(a, b)
. iu ny chng t rng G(f) tr mt trong R
2
.

nh l.
Cho f, g, h l cc a thc thuc R[x] tho mn cc iu kin
i) deg(f) = deg(g) + deg(h)
Vietnamese IMO Team Training Camp 2010
12 | Trn Nam Dng 6/2010

ii) deg(g) > deg(h) hoc deg(g) = deg(h) v g* + h* 0, trong g*, h*
tng ng l cc h s cao nht ca g v h.
Khi vi mi n nguyn dng, tn ti khng qu 1 a thc P(x) c bc n tho mn
iu kin
P(f) = P(g)P(h).

Chng minh:
Gi s P l a thc bc n tho mn phng trnh (1), deg(f) = f, deg(g) = g, deg(h) = h,
cc h s cao nht ca P, f, g, h tng ng l P*, f*, g*, h*. So snh h s cao nht hai v
ca cc a thc trong phng trnh
P(f(x))P(g(x)) = P(h(x))
ta c P*(f*)
n
.P*(g*)
n
= P*(h*)
n
t suy ra P* = (h*/f*g*)
n
.
Nh vy, nu gi s ngc li, tn ti mt a thc Q bc n (khc P) cng tho mn
phng trnh (1) th Q* = P* v ta c
Q(x) = P(x) + R(x) vi 0 r = deg(R) < n
(ta quy c bc ca a thc ng nht 0 bng -, do deg(R) 0 ng ngha R khng
ng nht 0)

Thay vo phng trnh (1), ta c
(P(f) + R(f))(P(g) + R(g)) = P(h) + R(h)
P(f)P(g) + P(f)R(g) + R(f)P(g) + R(f)R(g) = P(h) + R(h)
P(f)R(g) + R(f)P(g) + R(f)R(g) = R(h) (2)
By gi ta xt cc trng hp
i) deg(f) deg(g). Gi s f > g. Khi bc ca cc a thc v tri (2) ln lt
l nf + rg, rf + ng, rf + rg, v do nf + rg > rf + ng > rf + rg nn v tri c bc l
nf + rg. Trong khi v phi c bc l rh = r(f+g) < nf + rg. Mu thun.
ii) deg(f) = deg(g). Khi , hai a thc u tin v tri ca (2) cng c bc l nf
+ rg = ng + rf v c th xy ra s trit tiu khi thc hin php cng. Tuy nhin,
xt h s cao nht ca hai a thc ny, ta c h s ca x
nf + rg
trong a thc th
nht v th hai ln lt bng P*(f*)
n
R*(g*)
r
, R*(f*)
r
P*(g*)
n
. Nh th, bc ca
x
nf+rg
trong tng hai a thc bng
P*R*f*
r
g*
r
(f*
(n-r)
+g*
(n-r)
) 0 do f* + g* 0. Nh vy, bc ca v tri ca (2)
vn l nf + rg, trong khi bc ca v phi l rh = rf + rg < nf + rg. Mu thun.

nh l c chng minh hon ton.

Bi tp

18. Chng minh rng cc phng trnh sau y khng c nghim nguyn dng
a) 4xy x y = z
2
;
b) x
2
y
3
= 7.

19. Chng minh rng khng tn ti hm s f: N* N* tho mn cc iu kin:
a) f(2) = 3;
Vietnamese IMO Team Training Camp 2010
13 | Trn Nam Dng 6/2010

b) f(mn) = f(m)f(n) vi mi m, n thuc N*;
c) f(m) < f(n) vi mi m < n.

20. Hi c tn ti hay khng cc s nguyn x, y, u, v, t tha mn iu kin sau
x
2
+ y
2
= (x+1)
2
+ u
2
= (x+2)
2
+ v
2
= (x+3)
2
+ t
2
.

21. Chng minh nh l sau: Cho f, g, h l cc a thc khng hng tha mn iu kin deg(f) + deg(g) =
deg(h), Q l mt a thc cho trc. Khi , vi mi s nguyn dng n v s thc a, tn ti nhiu nht
mt a thc P tha mn ng thi cc iu kin sau: i) deg(P) = n, ii) P* = a iii) P(f)P(g) = P(h) + Q.

2. Quy np ton hc

Quy np ton hc l mt trong nhng nt c trng ca suy lun trong ton hc. T duy
quy np rt cn thit trong s hc, i s, t hp, hnh hc v gii tch, ni chung l trong
tt c cc lnh vc ca ton hc.

Quy np ton hc v bt ng thc

Gp cc bt ng thc c nhiu bin s, ta c th ngh ngay n php quy np ton hc.
D nhin, vic p dng quy np th no lun l c mt ngh thut.

V d 1. (Chng minh bt ng thc Cauchy bng quy np tin).
Cho a
1
, a
2
, , a
n
l cc s thc khng m. Chng minh rng ta lun c

n
n n
a a a n a a a ... ...
2 1 2 1
+ + +

Trong cc ti liu, bt ng thc ny thng c chng minh bng php quy np li,
hay quy np kiu Cauchy. y chng ta trnh by mt php chng minh khc.

C s quy np vi n = 1, 2 c kim tra d dng. Gi s bt ng thc c chng
minh cho n s. Xt n+1 s khng m a
1
, a
2
, , a
n+1
. t a
1
a
2
a
n+1
= A
n+1
. Nu tt c cc
s bng nhau th bt ng thc ng. Trong trng hp ngc li, phi tn ti hai s a
i
, a
j

sao cho a
i
< A < a
j
. Khng mt tnh tng qut, c th gi s a
n
< A < a
n+1
. Khi ta c (a
n

A)(a
n+1
A) < 0, suy ra a
n
+ a
n+1
> a
n
a
n+1
/A + A. T ta c
a
1
+ a
2
+ + a
n
+ a
n+1
> a
1
+ + a
n-1
+ a
n
a
n+1
/A + A (1)
By gi p dng bt ng thc Cauchy cho n s a
1
+ + a
n-1
+ a
n
a
n+1
/A ta c
nA
A
a a
a a a n a a a a
n
n n
n n n
= + + + +
+

1
1 2 1 1 2 1
... ...
Kt hp vi (1) ta c pcm.

V d 2. Cho n 2 v cho x
1
, x
2
, , x
n
l cc s thc thuc [0, 1]. Chng minh rng
x
1
(1-x
2
) + x
2
(1-x
3
) + + x
n
(1-x
1
) [n/2]

Vietnamese IMO Team Training Camp 2010
14 | Trn Nam Dng 6/2010

Vn bi ton ny l bc chng minh t n n+1 trong trng hp n chn l khng
th (do lc v phi khng thay i v ta cn chng minh phn thay i v tri nh
hn hay bng 0:
x
n
(1-x
n+1
) + x
n+1
(1-x
1
) x
n
(1-x
1
) 0
<=> x
n
(x
1
-x
n+1
) + x
n+1
(1-x
1
) 0
R rng biu thc v tri c th nhn c nhng gi tr dng v bc quy np ca chng
ta khng thc hin c.

Ta c th vt qua c kh khn ny nu thc hin bc quy np nhy cch, tc l t n
n+2. Khi , do [(n+2)/2] [n/2] = 1 nn ta cn chng minh:
x
n
(1-x
n+1
) + x
n+1
(1-x
n+2
) + x
n+2
(1-x
1
) x
n
(1-x
1
) 1.
iu ny tng ng vi
A = x
n
(x
1
-x
n+1
) + x
n+1
(1-x
n+2
) + x
n+2
(1-x
1
) 1.

Bt ng thc ny c th chng minh c kh d dng (chng ti dnh cho bn c).

Cui cng, ta cn chng minh c s quy np, trong trng hp ny l trng hp n = 2
v n = 3.
x
1
(1-x
2
) + x
2
(1-x
1
) 1
v
x
1
(1-x
2
) + x
2
(1-x
3
) + x
3
(1-x
1
) 1

Bt ng thc th nht ng do
x
1
(1-x
2
) + x
2
(1-x
1
) = 1 (1x
1
)(1x
2
) x
1
x
2
1
Bt ng thc th hai ng do
x
1
(1-x
2
) + x
2
(1-x
3
) + x
3
(1-x
1
) = 1 (1x
1
)(1x
2
)(1x
3
) x
1
x
2
x
3
.

Ch rng, trong php chng minh bt ng thc A 1 (php chng minh quy np) c
th s dng n bt ng thc x
1
(1-x
2
) + x
2
(1-x
3
) + x
3
(1-x
1
) 1.

V d 3. Cho n 2 v x
1
, x
2
, , x
n
l n s nguyn phn bit. Chng minh rng
(x
1
-x
2
)
2
+ (x
2
-x
3
)
2
+ + (x
n
x
1
)
2
4n 6

Ta th xt bc quy np t n n+1. Khi v phi thay i 4 n v, trong khi thay i
v tri l
A = (x
n
-x
n+1
)
2
+ (x
n+1
-x
1
)
2
(x
n
x
1
)
2

Ta cn chng minh A 4.

Nu nhn k li bt ng thc cn chng minh v cc iu kin rng buc th ta thy rng
bt ng thc A 4 ni chung khng ng ! Vy phi lm th no?

Ta vit li bt ng thc di dng
A = x
n+1
(2x
n+1
-x
n
-x
1
) + 2x
1
x
n

Vietnamese IMO Team Training Camp 2010
15 | Trn Nam Dng 6/2010

By gi, ta mi ch n hai tnh cht quan trng ca bt ng thc ban u
1) V tri khng thay i nu ta cng thm vo mi s hng x
i
mt i lng a c
nh. Do ta c th gi s x
n+1
= 0
2) y l bt ng thc hon v, do ta c th gi s x
n+1
=
min{x
1
,x
2
,,x
n
,x
n+1
}
T y suy ra A = 2x
1
x
n
2.2 = 4 (v x
1
, x
2
> 0 l cc s nguyn phn bit nn x1x2
1.2 = 2).

Bi ton c gii quyt.

V d 4. Cho cc s dng a
1
, a
2
, , a
n
tha mn iu kin a
1
+ a
2
+ + a
n
= n. Chng
minh rng ta c bt ng thc
) ... 1 (
) 1 ( 8 1
...
1 1
2 1
2
2 1
n
n
a a a
n
n
n
a a a

+ + + .
Gii.
Ta chng minh kt qu tng qut hn
) ... 1 (
1
...
1 1
2 1
2 1
n n
n
a a a m n
a a a
+ + +
vi mi .
) 1 ( 8
2
n
n
m
n


Vi n = 1, bt ng thc hin nhin ng. Gi s bt ng thc ng n n = k, ta
chng minh bt ng thc cng ng vi n = k+1. Tht vy, gi s a
k+1
= max{a
1
, a
2
, ,
a
k
}, suy ra . 1
...
1

+ +
=
k
a a
b
k
t b
i
= a
i
/b suy ra b
1
+ b
2
+ + b
k
= k. Ch l

2 2
1
1
1
1 1 1 1
1
1
1 ( 8
) 1 (
8
1 k
k
k
k
m
k
a kb
m a b m a b m
k
k
k
k k
k
k k
k
k

+
= = |

\
|
+
+

+
+
+
+ + + +
+
+

Do , s dng gi thit quy np ta c

1 1 1 1 1
1
1
1 1
1
1 1
1
1
1
...
1
...
1
)
...
1 (
1
...
1
) ... 1 (
1
...
1
+ + + +
+ +
+
+
+
+ + + +
+ +
+ +
k
k
k k k k
k
k
k
k
k
k
k
k k
k
k
k
a b m
b
k
a a a m
a a
b
a a
a b m
b
k
a a
b b a b m k
b b

Cui cng, ta phi chng minh
Vietnamese IMO Team Training Camp 2010
16 | Trn Nam Dng 6/2010


) ... 2 1 )( 1 (
) 1 (
)) 1 ( 1 ( ) 1 (
1
1
) 1 ( ) 1 (
1
0 1
1
1
1
1
1 1
1
1
1
1 1

+
+
+ +
+
+
+
+ +
+ + + +
+

+ +
+
+
+ +
+ +
k
k
k
k
k
k
k
k
k
k
k
k
k
kb b kb k b
k k
m
kb k b m k
kb k b
k
a b m k
a b
k
m k
a
a b m
b
k

Bt ng thc ny ng v
k
k
kb k b b
k
k
m
k
4
) 1 (
) 1 ( , 1 ,
) 1 (
8
2
2
1
+
+
+

+
.
Vy ta c iu phi chng minh.

Bi tp

1. Chng minh rng vi x
1
x
2
x
n
0 ta c bt ng thc


= =

n
i
i
n
i
i
i
x
x
1 1
2


2. Chng minh rng nu a
1
, a
2
, , a
n
l cc s nguyn dng phn bit th ta c bt ng thc


= =
|

\
|
+
n
i
n
i
i i i
a a a
1
2
1
3 5 7
2 ) (

3. (Bt ng thc Mc-Lauflin) Vi mi s thc a
1
, a
2
, , a
2n
v b
1
, b
2
, , b
2n
ta c bt ng thc


= = =

=
|

\
|
|

\
|

n
k
n
k
k k
n
k
k k k k
n
k
k k
b a b a b a b a
2
1
2
2
1
2
1
2 1 2 1 2 2
2
1
2 2
) (

4. Cho x
1
, x
2
, , x
n
l cc s thc dng. Chng minh rng

=
+ +

+
n
i
i i i
i
n
x x x
x
1
2 1
2
2
2

trong x
n+1
= x
1
, x
n+2
= x
2
.

5. Cho a
1
, a
2
, , a
n
l cc s thc dng tha mn iu kin a
1
+ a
2
+ + a
n
= n. Chng minh rng
(n-1)(a
1
2
+a
2
2
++a
n
2
) + na
1
a
2
a
n
n
2
.

6. Cho n 3 v a
1
, a
2
, , a
n
l cc s nguyn dng tha mn iu kin
i
i i
i
a
a a
b
1 1 +
+
= nguyn vi mi
i = 1, 2, , n ( y a
n+1
= a
1
, a
0
= a
n
). Chng minh rng khi ta c bt ng thc
2 3 2
1


=
n b n
n
i
i



Vietnamese IMO Team Training Camp 2010
17 | Trn Nam Dng 6/2010

Quy np trong s hc

Quy np c s dng rng ri trong s hc, c bit l trong cc bi ton v ng d, v
bc theo modulo m. Di y ta xem xt mt s v d kinh in.

nh l nh Fermat: Nu p l s nguyn t th a
p
a chia ht cho p vi mi a nguyn.

nh l ny c th chng minh bng php quy np ton hc, s dng tnh cht
k
p
C chia
ht cho p vi mi k = 1, 2, , p-1.

V d 4. (VMO 1997) Chng minh rng vi mi s nguyn dng n u chn c s
nguyn dng k 19
k
97 chia ht cho 2
n
.

Vi n = 1, n = 2 ta chn k = 2 nn ch cn xt vi n 3. Ta c nhn xt sau

n
n
t
n
. 2 1 19
2
2
=

vi t
n
l. (1)
Tht vy, vi n = 3, khng nh 1 ng. Gi s khng nh ng vi n. Khi
) ( 2 2 2 ) 1 19 )( 1 19 ( 1 19
1 2 2 2
2 2 1
n n
n
n
n
n
t s t s
n n n
+
= = + =

vi (s
n
t
n
) l.
Nhn xt c chng minh.

Ta chng minh bi ton bng quy np. Vi n = 3 ng. Gi s tn ti k
n
thuc N* sao cho
a
n k
n
. 2 97 19 =
Nu a chn th 97 19
n
k
chia ht cho 2
n+1
. Nu a l, t k
n+1
= k
n
+ 2
n-2
. Khi theo nhn
xt ta c
) 97 19 ( 2 ) 1 19 ( 97 ) 97 19 ( 19 97 19
2 2 2
1
2 2 2
n
n k k
t a
n n
n
n
n
+ = + =

+

chia ht cho 2
n+1
(pcm).

Bi tp

4. Chng minh rng vi mi s nguyn dng n s n! tho mn iu kin sau: vi mi c s ca n,
khc vi n! c th tm c mt c s khc ca n! sao cho tng hai c s li l c s ca n!.

5. Chng minh rng nu s nguyn dng N c th biu din di dng tng bnh phng ca ba s
nguyn chia ht cho 3 th n cng c th biu din di dng tng bnh phng ca ba s khng chia ht
cho 3.

6. Chng minh rng tn ti v s hp s n sao cho 3
n-1
2
n-1
chia ht cho n.

Quy np trong cc bi ton tr chi

Cc bi ton tr chi chnh l dng ton s dng n quy np ton hc nhiu nht. Ch
l quy np ton hc y bao gm hai phn: d on cng thc v chng minh cng
thc v trong rt nhiu trng hp, vic d on cng thc ng vai tr then cht.
Vietnamese IMO Team Training Camp 2010
18 | Trn Nam Dng 6/2010


V d 5. Hai ngi A v B cng chi mt tr chi. Ban u trn bn c 100 vin ko. Hai
ngi thay phin nhau bc ko, mi ln c bc k vin vi k {1, 2, 6} . Hi ai l
ngi c chin thut thng, ngi i trc hay ngi i sau?

Ta s khng bt u t 100 vin ko m bt u t nhng s ko nh hn. Gi s ban u
trn bn c n vin ko. Nu n = 1, 2, 6 th r rng ngi th nht c chin thut thng (ta
gi n gin l ngi th nht thng). Vi n = 3 th ngi th hai thng, bi ngi th
nht ch c th bc 1 hoc 2 vin v tng ng ngi th hai bc 2 hay 1 vin thng.
Vi n = 4 ngi th nht thng bng cch bc 1 vin ko v y ngi th hai vo th
thua. Tng t, vi n = 5 ngi th nht thng. Vi n = 7, ngi th hai thng v c ba
cch i c th ca ngi th nht (bc 1, 2, 6 vin) u dn n th thng cho ngi th
hai (tng ng cn 6, 5, 1 vin ko trn bn), n = 8 ngi th nht thng
Bng cch l lun tng t nh vy, ta lp c bng sau


n 1 2 3 4 5 6 7 8 9 10 11 12 13 14 15 16 17
KQ 1 1 2 1 1 1 2 1 1 2 1 1 1 2 1 1 2

T bng kt qu, c th d on c l ngi th nht s thng nu n c s d l 1, 2,
4, 5, 6 trong php chia cho 7, v ngi th hai s thng nu n c s d l 0, 3 trong php
chia cho 7.

Sau khi c d on ta tm cch chng minh cht ch d on ca mnh bng php quy
np ton hc. t n = 7k+r vi r = 1, 2, , 6, 7 ta chng minh d on trn bng quy np
theo k. Vi k = 0 mnh c kim chng qua bng trn.

Xt n = 7(k+1) + r vi r = 1, 2, , 6, 7
Nu r = 1, 2, 6, ngi th nht bc tng ng 1, 2, 6 vin a v trng hp trn bn
cn 7k+7 vin ko l th thua cho ngi th hai (theo gi thit quy np), v th ngi th
nht thng.
Nu r = 3, ngi th nht c 3 cch bc
+ Bc 1 vin, cn 7(k+1) + 2 l th thng cho ngi th hai (ta va chng minh
trn)
+ Bc 2 vin, cn 7(k+1) + 1, tng t cng l th thng cho ngi th hai
+ Bc 6 vin, cn 7k + 4 vin l th thng ca ngi th hai (theo gi thit quy
np).
Nh vy trng hp ny ngi th nht thua.
Nu r = 4, 5, ngi th nht bc tng ng 1, 2 vin a v trng hp 7(k+1) + 3 l
th thua cho ngi th hai, v v vy ngi th nht thng.
Cui cng, trng hp r = 7, ngi th nht c 3 cch bc
+ Bc 1 vin, cn 7(k+1) + 6 l th thng cho ngi th hai (chng minh trn)
+ Bc 2 vin, cn 7(k+1) + 5 l th thng cho ngi th hai (chng minh trn)
Vietnamese IMO Team Training Camp 2010
19 | Trn Nam Dng 6/2010

+ Bc 6 vin, cn 7(k+1) + 1 l th thng cho ngi th hai (chng minh trn)
Vy ngi th nht thua.

Nh th d on ca chng ta c chng minh hon ton.

V 100 chia 7 d 2 nn theo l lun trn th ngi th nht c chin thut thng.

V d 6. Cu b v Freken Bock cng chi mt tr chi. Trn bn c mt s ko. Bc i
u tin, cu b chia s ko thnh 3 ng khc rng, sau Freken chn ra 2 ng a
cho Carlson, ng cn li Freken li chia ra thnh 3 ng khc rng v cu b li chn
ra hai ng a cho Carlson, ng cn li chia thnh 3 ng khc rng Ai n lt
mnh khng i c na th thua. Hi ai l ngi c chin thut thng nu trn bn c:
a) 7 vin ko ;
b) 9 vin ko ;
c) 12 vin ko ;
d) 14 vin ko ;
e) Mt s ko bt k.

Bi tp

7. a) Trn bng c s 2010. Hai ngi A v B cng lun phin thc hin tr chi sau: Mi ln thc hin,
cho php xo i s N ang c trn bng v thay bng N-1 hoc [N/2]. Ai thu c s 0 trc l thng
cuc. Hi ai l ngi c chin thut thng, ngi i trc hay ngi i sau.
b) Cng cu hi vi lut chi thay i nh sau: Mi ln thc hin, cho php xo i s N ang c trn
bng v thay bng N-1 hoc [(N+1)/2].

8. C bng ch nht gm m x n . Hai ngi A v B cng lun phin nhau t mu cc ca bng, mi ln
t cc to thnh mt hnh ch nht. Khng c php t nhng t. Ai phi t cui cng l thua.
Hi ai l ngi c chin thut thng, ngi i trc hay ngi i sau?

9. An v Bnh chi tr on s. An ngh ra mt s no nm trong tp hp X = {1, 2, , 144}. Bnh c
th chn ra mt tp con bt k A ca X v hi S ca bn ngh c nm trong A hay khng ? . An s tr
li C hoc Khng theo ng s tht. Nu An tr li c th Bnh phi tr cho An 2.000 ng, nu An tr
li Khng th Bnh phi tr cho An 1.000 ng. Hi Bnh phi tt t nht bao nhiu tin chc chn tm
ra c s m An ngh ?

Quy np trong bi ton m

Xy dng cng thc truy hi l mt trong nhng phng php quan trng gii bi ton
m. T tng quy np y rt r rng: tm cng thc cho bi ton m vi kch
thc n, ta s dng kt qu ca bi ton m tng t vi kch thc nh hn.

V d 7. (Bi ton chia ko ca Euler)
Cho k, n l cc s nguyn dng. Tm s nghim nguyn khng m ca phng trnh
x
1
+ x
2
+ + x
n
= k (*)
Vietnamese IMO Team Training Camp 2010
20 | Trn Nam Dng 6/2010


Gii. Gi s nghim nguyn khng m ca phng trnh trn l S(n, k). D dng thy
rng S(1, k) = 1. tnh S(n, k), ta ch rng (*) tng ng vi
x
1
+ ...+ x
n-1
= k - x
n
(**)
Suy ra vi x
n
c nh th s nghim ca (**) l S(n-1, k-x
n
). T ta c cng thc
S(n, k) = S(n-1, k) + S(n-1, k-1) + ...+ S(n-1, 0)
y c th coi l cng thc truy hi tnh S(n, k). Tuy nhin, cng thc ny cha tht tin
li. Vit cng thc trn cho (n, k-1) ta c
S(n, k-1) = S(n-1, k-1) + S(n-1, k-2) + ...+ S(n-1, 0)
T y, tr cc ng thc trn v theo v, ta c
S(n, k) - S(n, k-1) = S(n-1, k)
Hay S(n, k) = S(n, k-1) + S(n-1, k)
T cng thc ny, bng quy np ta c th chng minh c rng S(n, k) = C
k
n+k-1
.

Trong nhiu trng hp, vic xt thm cc bi ton ph s gip chng ta thit lp nn cc
h phng trnh truy hi, t suy ra cng thc truy hi cho cc bi ton chnh.

V d 8. Xt tp hp E = {1, 2, , 2010}. Vi tp con A khc rng ca E, ta t
r(A) = a
1
a
2
+ + (-1)
k-1
a
k

trong a
1
, a
2
, , a
k
l tt c cc phn t ca A xp theo th t gim dn. Hy tnh tng

=
E A
A r S ) ( .
t E
n
= {1, 2, , n} v

=
n
E A
n
A r S ) ( . Xt S
n+1
, bng cch chia cc tp con ca E
n+1

thnh 2 loi, loi khng cha n+1 v cha n+1, ta c


+
+ = + + = + + = =
+ n n n n n
E A
n
E A E A E A E A
n
n A r n A r n A r A r A r S . 2 ) 1 ( )) ( 1 ( ) ( }) 1 { ( ) ( ) (
1
1


Ghi ch. y l tnh hung may mn c bit khi chng ta truy hi m khng truy hi,
ngha l ra c cng thc tng minh lun.

V d 9. C 2n ngi xp thnh 2 hng dc. Hi c bao nhiu cch chn ra mt s ngi
(t nht 1) t 2n ngi ny, sao cho khng c hai ngi no ng k nhau c chn.
Hai ngi ng k nhau l hai ngi c s th t lin tip trong mt hng dc hoc c
cng s th t hai hng.

Gi S
n
l s cch chn ra mt s ngi t 2n ngi xp thnh 2 hng dc v T
n
l s cch
chn ra mt s ngi t 2n-1 ngi xp thnh 2 hng dc, trong khuyt mt ch u
ca mt hng. Ta c S
1
= 2, T
1
= 1.

1 3
2 4

Hnh 1. S
n
vi n = 5
Vietnamese IMO Team Training Camp 2010
21 | Trn Nam Dng 6/2010


1 2


Hnh 2. T
n
vi n = 5

Xt 2n ngi xp thnh 2 hng dc (nh hnh 1). Ta xt cc cch chn tho mn iu
kin u bi. Xy ra cc kh nng sau :
1) Ngi v tr s 1 c chn : Khi ngi v tr s 2 v s 3 khng c
chn C T
n-1
+ 1 cch chn (+1 l do b sung cch chn khng chn g
c )
2) Ngi v tr s 2 c chn : Tng t, c T
n-1
+ 1 cch chn.
3) C hai ngi v tr s 1 v s 2 u khng c chn: C S
n-1
cch chn.
Vy ta c S
n
= S
n-1
+ 2T
n-1
+ 2 (1).
Xt 2n-1 ngi xp thnh 2 hng dc (nh hnh 2). Ta xt cc cch chn tho mn iu
kin u bi. Xy ra cc kh nng sau :
1) Ngi v tr s 1 c chn : Khi ngi v tr s 2 khng c chn
c T
n-1
+ 1 cch chn
2) Ngi v tr s 1 khng c chn : c S
n-1
cch chn.
Vy

ta c T
n
= S
n-1
+ T
n-1
+ 1 (2)

T (1) ta suy ra 2T
n-1
= S
n
S
n-1
2, 2T
n
= S
n+1
S
n
2. Thay vo (2), ta c
S
n+1
S
n
2 = 2S
n-1
+ S
n
S
n-1
2 + 2
S
n+1
= 2S
n
+ S
n-1
+ 2
T y d dng tm c

2
2 ) 2 1 ( ) 2 1 (
1 1
+ +
=
+ + n n
n
S
Bi tp

10. Tm s cch lt ng i kch thc 3 x 2n bng cc vin gch kch thc 1 x 2.
11. Tm s tt c cc b n s (x
1
, x
2
, , x
n
) sao cho
(i) x
i
= 1 vi i = 1, 2, , n.
(ii) 0 x
1
+ x
2
+ + x
r
< 4 vi r = 1, 2, , n-1 ;
(iii) x
1
+ x
2
+ + x
n
= 4.
12. Trn bn c 365 tm ba m trn mt p xung ca n c ghi cc s khc nhau. Vi 1.000 ng An c
th chn ba tm ba v yu cu Bnh sp xp chng t tri sang phi sao cho cc s vit trn chng c
xp theo th t tng dn. Hi An, b ra 2.000.000 c th chc chn sp xp 365 tm ba sao cho cc s
c vit trn chng c xp theo th t tng dn hay khng ?

13. (Bi ton con ch, IMO 1979) Gi A v E l hai nh i din ca mt bt gic. T mt nh bt k
ngoi tr E, con ch nhy n hai nh k. Khi n nhy n nh E th n ngng li. Gi a
n
l s cc
ng i khc nhau vi ng n bc nhy v kt thc ti E. Chng minh rng a
2n-1
= 0,
2
) 2 2 ( ) 2 2 (
1 1
2

+
=
n n
n
a .
Vietnamese IMO Team Training Camp 2010
22 | Trn Nam Dng 6/2010

Quy np v mt s nh l trong ti u t hp

nh l 1. (Hall, 1935) Cho th hai phe X, Y. Vi mi tp con A thuc X, gi G(A) l
tp cc nh thuc Y k vi mt nh no thuc A. Khi iu kin cn v tn
ti mt n nh f: X Y sao cho x k f(x) l |G(A)| |A| vi mi A khc rng thuc X.

Chng minh. iu kin cn l hin nhin: Nu tn ti n nh f th vi mi A = {x
1
, x
2
,
, x
r
} thuc X, ta c G(A) cha cc phn t phn bit f(x
1
), , f(x
r
), do |G(A)| r =
|A|.
Ta chng minh iu kin bng quy np theo |X|. Khi |X| = 1, khng nh l hin nhin.
Gi s nh l ng vi cc tp X vi |X| < n. Gi s by gi |X| = n. Ta xt hai trng
hp:
1) Gi s vi mi A X (A X), ta c |G(A)| > |A|. Chn mt phn t x
0
bt k thuc X,
theo iu kin |G({x
0
})| 1, do tn ti y
0
thuc Y k vi X. Ta t f(x
0
) = y
0
. By gi
xt X = X \{x} v Y = Y \ {y}, A X v G(A) l tp cc nh thuc Y k vi A. Khi
|G(A)| |G(A)| - 1 |A|. V |X| < |X| nn theo gi thit quy np, tn ti n nh f: X
Y sao cho f(x) k x vi mi x thuc x. B sung thm f(x
0
) = y
0
ta c n nh f: X
Y tha mn yu cu nh l.
2) Trong trng hp ngc li, tn ti A X (A X) sao cho |G(A)| = |A|. Khi , do |A|
< |X| nn tn ti n nh f: A G(A). Xt X = X \ A, Y = Y \ G(A). Xt B thuc X v
G(B) l tp cc nh thuc Y k vi B. Nu |G(B)| < |B| th ta c
|G(A B)| = |G(A)| + |G(B)| < |A| + |B| = |A B|
mu thun vi iu kin nh l. Nh vy ta c |G(B)| |B| vi mi B thuc X. Theo gi
thit quy np, tn ti n nh g: X Y sao cho g(x) k vi x. Nh vy, ta c th xy
dng c n nh h: X Y sao cho h(x) k vi x: c th h(x) = f(x) nu x thuc A v
h(x) = g(x) nu x thuc X \ A.

Quan h trn tp hp X c gi l mt quan h th t nu tha mn ng thi cc
iu kin sau:
i) x x vi mi x thuc X (tnh phn x)
ii) Nu x y, y x th x = y (tnh phn xng)
iii) Nu x y, y z th x z (tnh bc cu)
Mt tp hp m trn xc nh mt quan h th t c gi l mt tp sp th t.

Cho X l mt tp sp th t, hai phn t x v y thuc X c gi l so snh c nu x
y hoc y x. Trong trng hp ngc li, ta ni x v y khng so snh c.

Mt tp con C ca X c gi l mt xch nu hai phn t bt k thuc C u so snh
c. Mt tp con A ca X c gi l mt i xch nu hai phn t bt k thuc A u
khng so snh c.

Vietnamese IMO Team Training Camp 2010
23 | Trn Nam Dng 6/2010

Phn t x thuc X c gi l phn t cc i nu t x y suy ra y = x. Phn t x c
gi l cc tiu nu t y x suy ra y = x. Phn t x thuc X c gi l ln nht nu x y
vi mi y thuc X v c gi l nh nht nu x y vi mi y thuc X. Xch C c gi
l cc i nu nh khng tn ti mt xch C cha C vi |C| > |C|. Tng t ta nh
ngha i xch cc i.

nh l 2. (Dilworth 1950) Cho mt tp sp th t X. S phn t ln nht ca mt i
xch ca X bng s nh nht cc xch ri nhau hp thnh X.

Chng minh 1. Gi M = max{|A| | A l i xch} v m l s nh nht cc xch ri nhau
hp thnh X. Nh vy tn ti i xch A ca X cha M phn t. V mt xch ch cha
c nhiu nht 1 phn t ca 1 i xch nn r rng ta c m M.

Ta chng minh m M bng quy np theo |X|. Gi a l mt phn t cc i ca X v M l
kch thc ca i xch ln nht trong X = X \ {a}. Khi , theo gi thit quy np X l
hp ca M xch ri nhau C
1
, C
2
, , C
M
. Ta cn chng minh rng hoc X cha i xch
vi M+1 phn t, hoc X l hp ca M xch. By gi, mi i xch kch thc M (M-i
xch) trong X cha mt phn t t mi C
i
. Gi a
i
l phn t ln nht trong C
i
thuc vo
mt M-i xch no trong X. D dng thy rng A = {a
1
, a
2
, , a
M
} l mt i xch
(nu chng hn a
i
< a
j
th v a
j
thuc vo mt M-i xch no v i xch ny li cha
mt phn t b
i
ca C
i
nn theo tnh ln nht ca a
i
, ta c b
i
a
i
< a
j
iu ny mu thun v
b
i
v a
j
cng thuc mt i xch). Nu A {a} l mt i xch trong X th ta c pcm.
Trong trng hp ngc li, ta c a > a
i
vi i no . Khi K = {a} {x C
i
: x a
i
}
l mt xch trong X v khng c M-i xch trong X \ K (v ai l phn t ln nht ca C
i

tham gia trong cc i xch nh vy), v th X \ K l hp ca M-1 xch.

Chng minh 2. (Theo H. Tverberg 1967)
Hin nhin ta c m M.
Ta chng minh M m bng quy np theo |X|.
iu ny l hin nhin nu |X|=0.
Gi s C l xch cc i trong X.
Nu mi i xch trong X\C c nhiu nht M-1 phn t th xong.
Gi s {a
1
,, a
M
} l mt i xch trong P\C.
nh ngha S
-
= {x X: i [ x a
i
]}, S
+
{x X: i [ a
i
x]}
V C l cc i, phn t ln nht ca C khng nm trong S
-
.
Theo gi thit quy np, nh l ng vi S
-
.
V th, S
-
l hp ca M xch ri nhau S
-
1
, , S
-
M
, trong a
i
S
-
i
.
Gi s rng x S
-
i
v x > a
i
. Nu nh tn ti a
j
vi x a
j
, ta s c a
i
< x
a
j.
Mu thun. V vy a
i
l phn t ln nht trong S
-
i
, i=1,,M.
Lm tng t i vi S
+
i
, ta c ai l phn t nh nht trong S
+
i
.
Kt hp cc xch li ta c iu phi chng minh.
Vietnamese IMO Team Training Camp 2010
24 | Trn Nam Dng 6/2010

3. Nguyn l Dirichlet

Nguyn l Dirichlet dng c in thng c dng chng minh tn ti theo kiu
khng xy dng (non-constructive), tc l bit i tng tn ti nhng khng ch ra c
th.

Nguyn l Dirichlet trong s hc

Trong s hc, nguyn l Dirichlet thng lin quan n cc bi ton chia ht, nguyn t
cng nhau. V d cc bi ton kinh in sau.

V d 1. Chn ra n+1 s t 2n s nguyn dng u tin.
a) Chng minh rng trong cc s c chn, c hai s phn bit x, y nguyn t
cng nhau.
b) Chng minh rng trong cc s c chn, c hai s x > y m x chia ht cho y.

V d 2. Chng minh rng t n s nguyn bt k lun c th chn ra mt s hoc mt s
s c tng chia ht cho n.

V d 3. (nh l Fermat-Euler v tng hai bnh phng)
Chng minh rng nu p l s nguyn t dng 4k+1 th tn ti cc s nguyn a, b sao cho
p = a
2
+ b
2
.

Chng minh. V p c dng 4k+1 nn theo kt qu ca nh l phn u, tn ti s
nguyn N sao cho N
2
+ 1 chia ht cho p, hay ni cch khc, N
2
-1 (mod p). Xt cc s
dng x + Ny vi x, y l cc s nguyn thuc ]] [ , 0 [ p . C tt c
2
) 1 ] ([ + p s nh vy.
V p p > +
2
) 1 ] ([ nn theo nguyn l Dirichlet, tn ti hai cp s (x, y) (x, y) sao cho
x + Ny x + Ny (mod p). T y suy ra
x x N(y y) (mod p)
=> (x x)
2
N
2
(y y)
2
(mod p)
By gi, nh li rng N
2
- 1 (mod p), ta suy ra
(x x)
2
- (y y)
2
(mod p)
(x x)
2
+ (y y)
2
(mod p)
Cui cng, ch rng 0 < (x x)
2
+ (y y)
2
< 2p ta suy ra iu phi chng minh.

Ngoi k thut kinh in vi chung v th, ta c th s dng mt bin th ca nguyn l
Dirichlet nh sau:

Tnh cht. Nu A, B l cc tp hp tho mn iu kin |A| + |B| > |A B| th A B
0.

Sau y l mt p dng ca tnh cht ny.
Vietnamese IMO Team Training Camp 2010
25 | Trn Nam Dng 6/2010


V d 4. Chng minh rng nu p l s nguyn t dng 4k+3 th tn ti cc s nguyn x, y
sao cho x
2
+ y
2
+ 1 chia ht cho p.

Chng minh. t r
i
= i
2
mod p vi i = 1, 2, , (p-1)/2 v s
i
= 1 i
2
mod p, i = 1, 2, ,
(p-1)/2 th d dng chng minh c rng r
i
i mt phn bit v s
i
i mt phn bit.
Hn na, r
i
v s
i
u thuc {1, 2, , p-1}.
t A = {r
1
, , r
(p-1)/2
}, B = {s
1
, , s
(p-1)/2
} th |A| = |B| = (p-1)/2 v |A B| p-1. Xy
ra hai trng hp
Trng hp 1. Nu | A B | < p-1 th theo tnh cht nn trn, ta c A B , tc l tn
ti i, j sao cho r
i
= s
j
, tng ng vi i
2
- 1- j
2
(mod p) i
2
+ j
2
+ 1 chia ht cho p
(pcm).
Trng hp 2. Nu | A B | = p-1 th A B = v nh vy, cc s r
1
, r
2
, , r
(p-1)/2
, s
1
,
s
2
, , s
(p-1)/2
i mt phn bit v ta c
r
1
+ r
2
+ + r
(p-1)/2
+ s
1
+ s
2
+ + s
(p-1)/2
= 1 + 2 + + p-1 0 (mod p)
iu ny mu thun v theo nh ngha ca r
i
v s
i
, ta c
r
1
+ r
2
+ + r
(p-1)/2
+ s
1
+ s
2
+ + s
(p-1)/2
1
2
+ 2
2
+ + [(p-1)/2]
2
+ (-1-1
2
) + +
(-1 [(p-1)/2]
2
) -(p-1)/2 (mod p).
Vy trng hp 2 khng xy ra, v nh th ta ri vo trng hp 1. Ta c iu phi
chng minh.

Ghi ch. L lun A v B v khng B suy ra A c gi l Tam on lun ri.

Bi tp

1. Xt dy s Fibonacci xc nh bi F
1
= F
2
= 1, F
n+1
= F
n
+ F
n-1
vi mi n 2. Chng minh rng vi mi
s nguyn dng m > 1. Tn ti v s s hng ca dy s chia ht cho m.

2. T khong (2
2n
, 2
3n
) chn ra 2
2n-1
+1 s l. Chng minh rng trong cc s c chn, tn ti hai s m
bnh phng mi s khng chia ht cho s cn li.

3. a) Chng minh rng khng tn ti s nguyn dng n sao cho 10
n
+ 1 chia ht cho 2003.
b) Chng minh rng tn ti cc s nguyn dng m, n sao cho 10
m
+ 10
n
+ 1 chia ht cho 2003.

4. (Vietnam TST 2001) Dy s nguyn dng a
1
, a
2
, , a
n
, tho mn iu kin
1 a
n+1
a
n
2001 vi mi n = 1, 2, 3, Chng minh rng tn ti v s cp s p, q sao cho q > p v a
q

chia ht cho a
p
.







Vietnamese IMO Team Training Camp 2010
26 | Trn Nam Dng 6/2010

Nguyn l Dirichlet trong i s

Trong i s nguyn l Dirichlet c th hin qua tnh cht c bn sau: Nu trn on
[a, b] c n s thc x
1
, x
2
, , x
n
(n 2) th tn ti cc ch s i j sao cho |x
i
-x
j
| (b-
a)/(n-1).

V d 5. Gia 7 s thc bt k lun tm c 2 s x v y sao cho .
3
1
1
0
+

<
xy
y x
. Gi
cc s cho l a
1
, a
2
, , a
7
. Vi mi s thc a, tn ti s thuc khong (-/2, /2)
sao cho a = tg(). Gi s a
1
= tg(
1
), a
2
= tg(
2
), , a
7
= tg(
7
). Theo tnh cht nu trn,
trong 7 s
1
,
2
, ,
7
tn ti hai s c hiu khng vt qu /6. Gi s hai s ny l
v , trong > . Khi
.
3
1
6
) (
) ( ) ( 1
) ( ) (
0 = =
+

<




tg tg
tg tg
tg tg

Nh vy cc s x = tg() v y = tg() l cc s cn tm.
nh l Kronecker v s tr mt l mt nh l c nhiu ng dng trong gii tch, i s,
gii tch phc. Di y ta xt chng minh rt s cp ca nh l ny ( dng tng
ng)

nh l Kronecker. Nu l s v t th tp hp S ={ {n} | n N*} tr mt trong [0,
1].
Chng minh. Ta cn chng minh rng vi mi khong (a, b) [0, 1], tn ti s nguyn
dng n sao cho {n} (a, b).

Trc ht, ta tm s nguyn dng N sao cho N > 1/(b-a). By gi xt N+1 s {},
{2}, , {(N+1)} thuc on [0, 1]. Theo nguyn l Dirichlet, tn ti hai s {p},
{q} vi 1 p < q N+1 sao cho |{p} {q}| 1/N.

rng {q} {p} = q - [q] - p + [p] = (q-p) - [q] + [p]. Do nu {q}
{p} > 0 th {(q-p)} = {q} {p}, cn nu {q} {p} < 0 th {(q-p)} = 1
({q} {p})

Ta xt hai trng hp

Trng hp 1. 1/N > {q} {p} > 0. Khi t k = q p, ta tm c s nguyn
dng k sao cho 0 < {k} < 1/N.

By gi ta gn m l s nguyn dng nh nht sao cho m{k} > a th ta c (m-
1){k} a. Do m{k} a + {k} < a + 1/N < b. Suy ra m{k} (a, b) [0, 1].

Vietnamese IMO Team Training Camp 2010
27 | Trn Nam Dng 6/2010

Nhng mk = m[k] + m{k}. Do 0 < m{k} < 1 nn t y ta c {mk} = m{k}.
t n = mk, ta c {n} (a, b) (pcm).

Trng hp 2. 0 > {q} {p} > -1/N. Khi t k = q p, ta tm c s nguyn
dng k sao cho 1 1/N < {k} < 1. t = 1 - {k} th 0 < < 1/N. Chng minh
tng t nh trn, ta tm c s nguyn dng m sao cho m thuc (1-b, 1-b), tc l
1 b < m(1 - {k}) < 1 a
a < m{k} m + 1 < b
a < {m{k}} < b
Cui cng, ta c mk = m[k] + m{k} nn {mk} = {m{k}} nn t n = mk ta c
iu phi chng minh.

Mt tnh hung rt n gin khc ca nguyn l Diriclet li c nhng ng dng rt hiu
qu trong nhiu bi ton chng minh bt ng thc, c bit l cc bt ng thc c iu
kin. l ch sau: Vi m l mt s thc cho trc v n 3 s thc a
1
, a
2
, , a
n
bt k
th lun tm c hai s trong cc s ny nm cng mt pha i vi m.

Gi hai s l x v y th ta c bt ng thc hin nhin sau: (x-m)(y-m) 0, t xy +
m
2
m(x+y). Nh vy, ta so snh c hai i lng khng cng bc vi nhau. Sau
y chng ta s xem xt mt s v d p dng.

V d 6. Cho x, y, z l cc s thc dng tho mn iu kin x + y + z + 1 = 4xyz. Chng
minh rng
xy + yz + zx x + y + z.

tng u tin ta ngh n l x l iu kin bng php th
1 4
1

+ +
=
yz
y x
z vo bt ng
thc cn chng minh, vit li thnh
xy y x y x
xy
y x
+ +

+ +
) 1 (
1 4
1

n y, ng li chng minh hnh thnh. V tri r rng 1 vi mi x, y cn v
phi s 1 nu x, y nm cng pha nhau i vi 1. Nhng iu ny, theo nhn xt n
gin trn ta lun c th chn c.

V d 7. Cho a, b, c l cc s thc khng m. Chng minh rng
(a
2
+ 2)(b
2
+ 2)(c
2
+ 2) 3(a + b + c)
2


p dng bt ng thc CBS, ta c
(a + b + c)
2
(a
2
+ 1 + 1)(1 + b
2
+ c
2
) = (a
2
+ 2)(b
2
+ c
2
+ 1). Nh vy ta ch cn cn
chng minh
(b
2
+ 2)(c
2
+ 2) 3(b
2
+ c
2
+ 1)
Vietnamese IMO Team Training Camp 2010
28 | Trn Nam Dng 6/2010

(b
2
1)(c
2
1) 0
iu ny lun c c nu ta chn b
2
, c
2
cng pha nhau i vi 1.

Bi tp

5. Cho a, b, c > 0, x = a + 1/b, y = b + 1/c, z = c + 1/a. Chng minh rng
xy + yz + zx 2(x+y+z)

6. (USA MO 2001) Cho a, b, c l cc s thc khng m tho mn iu kin a
2
+ b
2
+ c
2
+ abc = 4. Chng
minh rng
0 ab + bc + ca abc 2.

7. Vi i = 1, 2, , 7 cc s a
i
, b
i
l cc s thc khng m tho mn iu kin a
i
+ b
i
2. Chng minh
rng tn ti cc ch s i j sao cho |a
i
a
j
| + |b
i
b
j
| 1.

8. (VMO 1996) Cho x, y, z l cc s thc dng tho mn iu kin xy + yz + zx + xyz = 4. Chng minh
rng
x + y + z xy + yz + zx.

9. Tm s thc k nh nht sao cho xyz + 2 + k[(x-1)
2
+ (y-1)
2
+ (z-1)
2
] x + y + z vi mi x, y, z > 0.

Nguyn l Dirichlet trong t hp

T hp l mnh t mu m nht cho cc phng php v k thut chng minh. V
nguyn l Dirichlet khng phi l mt ngoi l. Trong t hp, mt c im c trng l
s bng n t hp ca cc trng hp, v vy, nguyn l Dirichlet cng vi cc nguyn l
khc nh nguyn l cc hn, nguyn l bt bin chnh l nhng cng c quan trng
chng ta nh hng trong bin cc trng hp.

Nguyn l Dirichlet thng c s dng trong cc bi ton th, t mu, cc bi ton
v thi u th thao ( th c hng), quen nhau ( th v hng).

V d 8. Trong mt gii bng chuyn c 8 i tham gia, thi u vng trn 1 lt. Chng
minh rng tm c 4 i A, B, C, D sao cho A thng B, C, D, B thng C, D v C thng
D.

Trong bng chuyn khng c ho, do 8 i thi u vng trn 1 lt th s c tt c 28
trn thng. Theo nguyn l Dirichlet, tn ti i bng A c t nht 4 trn thng. Xt 4 i
thua A. 4 i ny u vi nhau 6 trn, do tn ti mt i thng t nht 2 trn (trong s
cc trn u gia 4 i ny vi nhau). Gi s l B v C, D l 2 i thua B. Cui cng,
nu C thng D th A, B, C, D l 4 i cn tm, cn nu D thng C th 4 i cn tm l A,
B, D, C.

Bi ton Ramsey l mt trong nhng bi ton kinh in m nhng trng hp c s ca
n rt th v v ph hp vi mc ton s cp.
Vietnamese IMO Team Training Camp 2010
29 | Trn Nam Dng 6/2010

V d 9. Chng minh rng trong mt nhm 6 ngi bt k c 3 ngi i mt quen nhau
hoc 3 ngi i mt khng quen nhau.

Hai ngi bt k c th quen nhau hoc khng quan nhau. Xt mt ngi A bt k th c
5 ngi khc hoc quen A hoc khng quen A. Do , theo nguyn l Dirichlet, phi xy
ra mt trong hai trng hp:
1) A quen t nht 3 ngi
2) A khng quen t nht 3 ngi
Ta xt trng hp th nht: A quen vi t nht 3 ngi, chng hn B, C, D. Nu trong 3
ngi B, C, D c t nht mt cp quen nhau, chng hn B quen C th ta c A, B, C i
mt quen nhau. Trong trng hp ngc li, ta c B, C, D i mt khng quen nhau.

Trng hp th hai c xt hon ton tng t. Bi ton c chng minh.

V d 10. Trong mt nhm gm 2n+1 ngi vi mi n ngi tn ti mt ngi khc n
ngi ny quen vi tt c h. Chng minh rng trong nhm ngi ny c 1 ngi quen
vi tt c mi ngi.

Ta chng minh rng trong nhm ngi ny c n+1 ngi i mt quen nhau. R rng c
2 ngi quan nhau v nu nh c k ngi i mt quen nhau (trong k n) th tn ti
mt ngi khc trong s h quen vi k ngi ny. T suy ra tn ti n+1 ngi i mt
quen nhau A
1
, A
2
, , A
n+1
.

Xt n ngi cn li. Theo iu kin,tn ti mt ngi A
i
quen vi tt c n ngi ny.
Nhng khi A
i
quen vi tt c mi ngi.

B quyt thnh cng ca nguyn l Dirichlet chnh l k thut xy chung v to th.
Trong nhiu bi ton, chung l g, th l g kh r rng, nhng trong nhiu bi ton, xy
chung v to th l c mt s tinh t. Ta phi bit chn cc thnh phn chnh v
hng n mc tiu.

V d 11. Cc s t 1 n 200 c chia thnh 50 tp hp. Chng minh rng trong mt
cc tp hp c ba s l di 3 cnh ca mt tam gic.

Thot nhn bi ton c v kh ri. Nhng nu rng vi 0 < a < b < c th iu kin
cn v a, b, c l di ba cnh ca mt tam gic l a + b > c th bi ton tr nn
n gin hn. R rng nu ch xt cc s t 100 n 200 th ba s bt k u l di 3
cnh ca 1 tam gic (a + b 100 + 101 = 201 > c). T ch cn xt 101 con th l cc
s t 100 n 200 ri p dng nguyn l Dirichlet cho 50 ci chung tp hp l xong.
y, r rng cc s t 1 n 99 ch c tc dng gy nhiu.

V d 12. Trn bn c quc t c 8 qun xe, i mt khng n nhau. Chng minh rng
trong cc khong cch i mt gia cc qun xe, c hai khong cch bng nhau. Khong
Vietnamese IMO Team Training Camp 2010
30 | Trn Nam Dng 6/2010

cch gia hai qun xe bng khong cch gia tm cc vung m qun cc qun xe
ng.

Trc ht ta m hnh ho bi ton. rng khong cch gia (p, q) v (m, n) bng
(p-m)
2
+ (q-n)
2
. Ta cn chng minh rng nu p
1
, p
2
, , p
8
l mt hon v ca (1, 2, 3, ,
8) th tn ti cc tp ch s {m, n} {p, q} sao cho (m-n)
2
+ (p
m
-p
n
)
2
= (p-q)
2
+ (p
p
p
q
)
2
.

8 qun xe to ra 28 khong cch. Nhng nu ta tm 2 khong cch bng nhau gia c 28
qun xe ny th ta s gp kh khn. Ta gii hn trong vic tm cc cp ch s dng {n,
n+1}. C 7 cp nh vy. Khi , ta ch cn tm n m sao cho (p
n+1
-p
n
)
2
= (p
m+1
-p
m
)
2
. V
1 p
i
8 nu (p
n+1
-p
n
)
2
ch c th l 1 trong 7 gi tr 1
2
, 2
2
, , 7
2
. V th ch c th xy
ra hai trng hp.

Trng hp 1. Tn ti n m sao cho (p
n+1
-p
n
)
2
= (p
m+1
-p
m
)
2
. Khi cc cp qun xe ti
(n, p
n
), (n+1, p
n+1
) v (m, p
m
), (m+1, p
m+1
) l cc cp xe cn tm.

Trng hp 2. Cc s (p
n+1
-p
n
)
2
i mt phn bit. Khi tn ti n sao cho (p
n+1
-p
n
)
2
=
4.

Lc ny, xoay hng thnh ct, ta li i n vic hoc tn ti n m sao cho (q
n+1
-q
n
)
2
=
(q
m+1
-q
m
)
2
hoc tn ti k sao cho (q
k+1
-q
k
)
2
= 2
2
. Trong trng hp th nht, bi ton c
gii quyt tng t nh trng hp 1 trn. Trong trng hp th
hai, cc qun xe ti (n, p
n
), (n+1, p
n+1
) v (q
k+1
,k+1), (q
k
, k) l cc cp xe cn tm.

Bi tp

10. Cc s 1, 2, 3, , 100 c th l thnh vin ca 12 cp s nhn no c khng?

11. Trong mt a gic li c cha khng t hn m
2
+1 im nguyn. Chng minh rng trong a gic li
ny tm c m+1 im nguyn cng nm trn mt ng thng.

12. Chng minh rng trong 9 ngi bt k, hoc c 3 ngi i mt quen nhau, hoc c 4 ngi i mt
khng quen nhau.

13. Chn ra 69 s nguyn dng t tp hp E = {1, 2, , 100}. Chng minh rng tn ti 4 s a < b < c
< d trong 4 s c chn sao cho a + b + c = d. Kt lun bi ton cn ng khng nu ta thay 69 bng 68?

14. Cc vung ca bng 100 x 100 c t bng 4 mu sao cho trn mi hng v trn mi ct c ng
25 c cng mt mu. Chng minh rng tn ti 2 dng v 2 ct sao cho bn nm giao ca chng
c t khc mu.

15. (Bulgarian MO 2005) Trong 9 ngi khng c 4 ngi no i mt quen nhau. Chng minh rng c
th chia 9 ngi ny thnh 4 nhm sao cho trong mi nhm khng c ngi no quen nhau.


Vietnamese IMO Team Training Camp 2010
31 | Trn Nam Dng 6/2010



Nguyn l Dirichlet trong hnh hc

Trong hnh hc, nguyn l Dirichlet thng c s dng trong cc bi ton lin quan
n di cnh, din tch, ln ca gc, cc bi ton trn li nguyn. y chng ti
ch gii hn trong vic gii thiu mt ng dng p ca nguyn l Dirichlet v chng
hnh trong hnh hc v mt s bi tp.

nh l Minkowsky l mt v d rt th v v ng dng ca hnh hc trong l thuyt s.
Chng ta bt u t mt kt qu rt n gin nhng hu ch

B 1. Trn mt phng cho hnh F c din tch ln hn 1. Khi tn ti hai im A, B
thuc F, sao cho vc t AB c ta nguyn.

Chng minh: Li nguyn ct hnh G thnh cc mu nh. Chng cc mu ny ln nhau,
do tng din tch ca cc mu ln hn 1, nn c t nht 2 mu c im chung (y chnh
l mt bin th ca nguyn l Dirichlet). Gi A, B l hai im nguyn thu ng vi im
chung ny th A, B l hai im cn tm.

B 2. (B Minkowsky) Trn mt phng cho hnh li F nhn gc ta lm tm i
xng v c din tch ln hn 4. Khi n cha mt im nguyn khc gc ta .

Chng minh: Xt php v t tm O, t s 1/2 , bin F thnh G. Do G c din tch ln
hn 1 nn theo b 1, tn ti hai im A, B thuc G sao cho vc-t AB c to
nguyn. Gi A l im i xng vi A qua O. Do hnh G i xng qua gc to nn
A thuc G. Do G li nn trung im M ca AB thuc G. Gi N l im i xng ca O
qua M th N thuc F v ON = AB, suy ra N l im nguyn khc O (pcm).

nh l 3. (nh l Minkowsky) Cho a, b, c l cc s nguyn, trong a > 0 v ac - b
2

= 1. Khi phng trnh ax
2
+ 2bxy + cy
2
= 1 c nghim nguyn.

Bi tp

15. Vi gi tr no ca n tn ti n im M
1
, M
2
, , M
n
sao cho tt c cc gc M
i
M
j
M
k
u khng t?

16. Cho 9 im nm trong hnh vung cnh 1. Chng minh rng tn ti mt tam gic c nh ti cc im
cho c din tch khng vt qu 1/8.

c thm bi Nguyn l chung v th




Vietnamese IMO Team Training Camp 2010
32 | Trn Nam Dng 6/2010

4. m bng hai cch

m bng hai cch l mt k thut thng dng to ra cc phng trnh, ng thc, cc
mi lin h gip chng ta gii quyt cc bi ton phng trnh, tnh ton hnh hc,
phng trnh hm, bt ng thc v c bit l cc bi ton t hp, trong c bi ton
m.

m bng hai cch v phng trnh hm

(Xem bi Gii phng trnh hm bng cch lp phng trnh)

m bng hai cch trong chng minh bt ng thc

Trong chng minh bt ng thc, m bng hai cch thng c s dng x l mt
s tng. Di y ta xem xt hai v d.

V d 1. Gi d l hiu gia s ln nht v s nh nht gia n s thc x
1
, x
2
, , x
n
(n 2). Chng minh rng ta c bt ng thc

<

n j i
j i
d n
x x d n
1
2
4
| | ) 1 (
Gii.
Khng mt tnh tng qut, gi s x
1
x
2
x
n
v t d
i
= x
i+1
- x
i
vi i = 1, 2, , n.
th d
i
0 v d = d
1
+ d
2
+ + d
n-1
. Vi i < j ta c |x
i
x
j
| = d
i
+ + d
j-1

Mi d
k
s c mt trong tng d
i
+ + d
j-1
nu i k v j-1 k, tc l trong k(n-k) tng.
T suy ra


<

=
=
n j i
n
k
k j i
d k n k x x
1
1
1
) ( | |
By gi ch n-1 k(n-k) n
2
/4 vi mi k = 1, 2, , n-1, ta suy ra bt ng thc cn
chng minh.

V d 2. Trong cc ca hnh vung 10 x 10 ta in cc s t 1 n 100 nh sau: hng
th nht in cc s t 1 n 10 t tri sang phi, hng th hai in cc s t 11 n 20
t tri sang phi An d nh ct hnh vung thnh cc hnh ch nht 1 x 2, ly tch ca
hai s trn mt hnh ch nht v cng tt c cc kt qu thu c vi nhau. An mun
cho tng thu c l nh nht c th. Hi An phi ct th no t c iu .
Gii.
Gi (a
i
, b
i
) i = 1, 2, , 50 l b s cc m An d nh ct. Ta c

( )
( )

= = = =
+ =
+
=
50
1
50
1
2
50
1
2 2
50
1
2 2 2
2
1
) (
2
1
2
i i
i i
i
i i
i
i i i i
i i
b a b a
b a b a
b a
Tng

=
+
50
1
2 2
) (
i
i i
b a bng tng bnh phng cc s t 1 n 100, do khng i. Nh vy
Vietnamese IMO Team Training Camp 2010
33 | Trn Nam Dng 6/2010

=
50
1 i
i i
b a t gi tr nh nht khi ( )

50
1
2
i
i i
b a ln nht. Nhng a
i
b
i
= 1 nu hai ny nm
trn mt hng v a
i
b
i
= 10 nu hai ny nm trn mt ct. T suy ra An phi ct
thnh cc dc th tng thu c s nh nht.
Bi tp

1. Cc hm s f(x) v g(x) xc nh trn tp hp cc s nguyn, c tr tuyt i khng vt qu 1000.
Gi m l s cc cp (x, y) sao cho f(x) = g(y), n l s cc cp (x, y) sao cho f(x)=f(y), v k l s cc cp x,
y sao cho g(x)=g(y). Chng minh rng 2m

n + k.

2. (IMO 2003) Cho n > 2 v cc s thc x
1
x
2
... x
n
, chng minh rng

= =

|
|

\
|

n
j i
j i
n
j i
j i
x x n x x
1 ,
2 2
2
1 ,
) ( ) 1 (
3
2
| | .
Chng minh rng du bng xy ra khi v ch khi dy s l cp s cng.

2
n
C trong cc bi ton t hp

Trong t hp
2
n
C l s cc cp to thnh t n phn t, l s cnh ca th y bc n.
Trong nhiu bi ton, s dng ngha t hp ny cng vi cch m bng hai cch gip
chng ta tm ra cha kho cho li gii.

V d 3. (Bulgarian MO 2006) Mt quc gia c 16 thnh ph v c 36 tuyn bay ni gia
chng. Chng minh rng ta c th t chc mt chuyn bay vng quanh gia 4 thnh ph.

Gii. Bi ton c th dch sang ngn ng th nh sau: Mt th G n c 16 nh v
36 cnh th c mt chu trnh di 4. Ta chng minh bng phn chng. Gi s trong
th khng c chu trnh di 4.

Vi mi nh x thuc G, gi d(x) l bc ca nh x. Khi s cp cc nh m c hai
cng k vi x bng
2
) ( x d
C . Ch rng mi mt cp c tnh nhiu nht mt nh x,
v nu khng ta s c mt chu trnh di 4.

S dng ng thc 72 ) ( =

G x
x d v bt ng thc Cauchy-Schwarz, ta c


= = |

\
|
= =
G x G x G x G x G x
x d
x d x d
x d x d
C C 126 36
32
72
) (
2
1
) (
32
1
2
) (
2
) (
120
2
2
2
2
) (
2
16
,
mu thun.


V d 4. (IMO ) Trong mt k thi c a th sinh v b gim kho, trong b 3 l mt s
nguyn l. Mi mt gim kho s nh gi th sinh l u hay rt. Gi s k l mt s
Vietnamese IMO Team Training Camp 2010
34 | Trn Nam Dng 6/2010

nguyn sao cho vi hai gim kho bt k th nh gi ca h trng nhiu nht k th
sinh. Chng minh rng k/a (b-1)/2b.

Gii.
Ta m s N cc b ba (GK, GK, TS) trong hai gim kho l khc nhau v nh gi
th sinh ging nhau. C b(b-1)/2 cp gim kho v mi mt cp gim kho nh vy nh
gi trng nhau ti khng qu k th sinh, do N kb(b-1)/2.
By gi xt mt th sinh X v m cc cp gim kho nh gi X ging nhau. Gi s c x
gim kho cho X u, th s c x(x-1)/2 cp cng cho X u v (b-x)(b-x-1)/2 cp cng
cho X rt, nh vy c (x(x-1) + (b-x)(b-x-1))/2 cp nh gi X ging nhau. Nhng (x(x-
1) + (b-x)(b-x-1))/2 = (2x
2
- 2bx + b
2
- b)/2 = (x - b/2)
2
+ b
2
/4 - b/2 b
2
/4 - b/2 = (b -
1)
2
/4 - 1/4. V (b - 1)
2
/4 l s nguyn (do b l), nn s cp nh gi X ging nhau s t
nht l (b - 1)
2
/4. T suy ra N a(b - 1)
2
/4. Kt hp hai bt ng thc li ta c
k/a (b - 1)/2b.
Bi tp

3. Trong Duma quc gia c 1600 i biu, lp thnh 16000 y ban, mi y ban c 80 i biu. Chng
minh rng c t nht hai y ban c khng di 4 thnh vin chung.

4. Sau khi khai trng c ng 10 ngy, mt nhn vin th vin cho bit :
1) Mi ngy c ng 8 ngi n c sch ;
2) Khng c ngi no n th vin 1 ngy qu 1 ln ;
3) Trong hai ngy bt k ca 10 ngy th c t nht l 15 ngi khc nhau cng n th vin.
Cn c ng thi c 3 iu kin m nhn vin th vin cung cp hy cho bit s ngi ti thiu
n th vin trong 10 ngy ni trn l bao nhiu ?

5. 2n k th thi u vng trn hai lt, mi lt hai k th bt k thi u vi nhau ng mt trn (thng
c 1 im, thua 0 im v ha 1/2 im). Ngi ta nhn thy rng lt hai, tng im ca mi k th
thay i khng nh hn n. Chng minh rng cc thay i ny ng bng n.

6. (Bulgarian MO 2003) Trong mt nhm n ngi c 3 ngi i mt quen nhau v mi mt ngi ny
quen nhiu hn 1 na s ngi trong nhm. Tm s t nht c th s b ba ngi i mt quen nhau.

m bng hai cch v mt s nh l trong l thuyt ti u t hp

Cho F l h cc tp con ca X. Vi x thuc x, ta gi d(x) l s phn t ca F cha x.

nh l 1. Cho F l h cc tp con ca tp hp X. Khi

Chng minh. Xt ma trn k M = (m
x,A
) ca F. Ngha l M l ma trn 0-1 vi |X| dng
nh s bi cc im x X v |F| ct nh s bi tp A F sao cho m
x,A
= 1 khi v ch
Vietnamese IMO Team Training Camp 2010
35 | Trn Nam Dng 6/2010

khi x A. rng d(x) bng s s 1 trn dng th x cn |A| l s s 1 trn ct th A.
Nh vy c v tri v v phi u biu din s s 1 ca M.
Nu ta xt th G = (V, E) trn tp nh V nh mt h cc tp con 2 phn t ca V th ta
c nh l Euler.

nh l 2. (Euler, 1736) Trong mi th, tng bc cc nh ca n bng hai ln s cnh
ca n v nh th, lun l mt s chn.
nh l sau c th c chng minh bng cch tng t

nh l 3. vi mi Y X.

(Hai tng ng thc u ng vi s s 1 trn cc hng Y. Cc tng ng thc th hai
m s ln xut hin ca x trong cc tp c dng A B).
Trng hp c bit khi F = E l tp con 2 phn t, ta c

nh l 4. Vi th G = (V, E), ta c


(Xem thm bi Cc bi ton ti u v h cc tp hp)

5. Nguyn l cc hn

Mt tp hp hu hn cc s thc lun c phn t ln nht v phn t nh nht. Mt tp
con bt k ca N (hoc N
k
) lun c phn t nh nht. Nguyn l n gin ny trong
nhiu trng hp rt c ch cho vic chng minh. Hy xt trng hp bin! l khu
quyt ca nguyn l ny.

Mt s v d m u

Ta xem xt mt s v d s dng nguyn l cc hn

V d 1. C 3 trng hc, mi trng c n hc sinh. Mi mt hc sinh quen vi t nht
n+1 hc sinh t hai trng khc. Chng minh rng ngi ta c th chn ra t mi trng
mt bn sao cho ba hc sinh c chn i mt quen nhau.

Gii.
Gi A l hc sinh c nhiu bn nht mt trng khc. Gi s ny l k. Gi s A
trng 1 v nhng bn quen A l B
1
, B
2
, ..., B
k
trng 2. Ta c .
2
1 +

n
k Cng theo gi
thit, c t nht 1 hc sinh C trng 3 quen vi A. Gi s C khng quen vi B
i
vi mi
i=1, 2, ..., k th C quen vi nhiu nht n - k hc sinh ca trng 2. Suy ra C quen vi t
nht n+1 - (n-k) = k+1 hc sinh trng 1, iu ny mu thun vi cch chn A. Vy C
phi quen vi mt B
i
no . Khi A, B
i
v C chnh l 3 hc sinh cn tm.
Vietnamese IMO Team Training Camp 2010
36 | Trn Nam Dng 6/2010


V d 2. Chng minh rng khng tn ti s n l, n > 1 sao cho 15
n
+ 1 chia ht cho n

Gii. Gi s tn ti mt s nguyn l n > 1 sao cho 15
n
+ 1 chia ht cho n. Gi p l c s
nguyn t nh nht ca n, khi p l. Gi s k l s nguyn dng nh nht sao cho 15
k

1 chia ht cho p.

V 15
2n
1 = (15
n
-1)(15
n
+1) chia ht cho p. Mt khc, theo nh l nh Fermat th 15
p-1

1 chia ht cho p. Theo nh ngha ca p, suy ra k l c s ca cc s p-1 v 2n. Suy ra k |
(p-1, 2n). Do p l c s nguyn t nh nht ca n nn (n, p-1) = 1. Suy ra (p-1, 2n) = 2.
Vy k | 2. T k = 1 hoc k = 2. C hai trng hp ny u dn ti p = 7. Nhng iu
ny mu thun v 15
n
+ 1 lun ng d 2 mod 7

Bi tp

1. Cho n im xanh v n im trn mt phng, trong khng c 3 im no thng hng. Chng minh
rng ta c th ni 2n im ny bng n on thng c u mt khc mu sao cho chng i mt khng
giao nhau.

2. Trn ng thng c 2n+1 on thng. Mi mt on thng giao vi t nht n on thng khc. Chng
minh rng tn ti mt on thng giao vi tt c cc on thng cn li.

3. Trong mt phng cho n > 1 im. Hai ngi chi ln lt ni mt cp im cha c ni bng mt
vc-t vi mt trong hai chiu. Nu sau nc i ca ngi no tng cc vc t v bng 0 th ngi
th hai thng; nu cho n khi khng cn v c vc t no na m tng vn cha c lc no bng 0 th
ngi th nht thng. Hi ai l ngi thng cuc nu chi ng?


Nguyn l cc hn v bt ng thc

Nguyn l cc hn thng c p dng mt cch hiu qu trong cc bt ng thc c
tnh t hp, dng chng minh tn ti k s t n s tha mn mt iu kin ny .

V d 1. (Moscow MO 1984) Trn vng trn ngi ta xp t nht 4 s thc khng m c
tng bng 1. Chng minh rng tng tt c cc tch cc cp s k nhau khng ln hn .
Gii.
Ta cn chng minh rng vi mi n 4 s thc khng m
1
, ...,
n
, c tng bng 1, ta c
bt ng thc
a
1
a
2
+ a
2
a
3
+ ... + a
n - 1
a
n
+ a
n
a
1
1/4.
Vi n chn n (n = 2m) iu ny c th chng minh d dng: t a
1
+ a
3
+ ... + a
2m - 1
= a;
khi , r rng,
a
1
a
2
+ a
2
a
3
+ ... + a
n - 1
a
n
+ a
n
a
1
(a
1
+ a
3
+ ... + a
2m1
) (a
2
+ a
4
+ ... + a
2m
) = a(1 a)
1/4.
Vietnamese IMO Team Training Camp 2010
37 | Trn Nam Dng 6/2010

Gi s n l v a
k
l s nh nht trong cc s cho. ( thun tin, ta gi s 1 < k < n
1 iu ny khng lm mt tnh tng qut khi n 4.) t b
i
=
i
, vi i = 1,..., k 1, b
k

= a
k
+ a
k + 1
v b
i
= a
i + 1
vi i = k + 1,..., n 1. p dng bt ng thc ca chng ta cho
cc s b
1
,..., b
n - 1
, ta c:
a
1
a
2
+ ... + a
k - 2
a
k - 1
+ (a
k - 1
+ a
k + 2
) b
k
+ a
k + 2
a
k + 3
+ ... + a
n - 1
a
n
+ a
n
a
1
1/4.
Cui cng, ta s dng bt ng thc
a
k - 1
a
k
+ a
k
a
k + 1
+ a
k + 1
a
k + 2
a
k - 1
a
k
+ a
k - 1
a
k + 1
+ a
k + 1
a
k + 2
(a
k - 1
+ a
k + 2
) b
k
.
suy ra iu phi chng minh.
nh gi trn y l tt nht; du bng xy ra khi 2 trong n s bng 1/2, cn cc s cn
li bng 0.

V d 2. Cho n 4 v cc s thc phn bit a
1
, a
2
, , a
n
tho mn iu kin
. 1 , 0
1
2
1
= =

= =
n
i
i
n
i
i
a a
Chng minh rng tn ti 4 s a, b, c, d thuc {a
1
, a
2
, , a
n
} sao cho
.
1
3
nabd d b a a nabc c b a
n
i
i
+ + + + + +

=


V d 3. Tng bnh phng ca mt 100 s thc dng ln hn 10000. Tng ca chng
nh hn 300. Chng minh rng tn ti 3 s trong chng c tng ln hn 100.

Gii. Gi s 100 s l C
1
C
2
... C
100
> 0. Nu nh C
1
100, th C
1
+ C
2
+ C
3
>
100. Do ta c th gi s rng C
1
< 100. Khi 100 - C
1
> 0, 100 - C
2
> 0, C
1
- C
2
0
C
1
- C
3
0, v vy
100(C
1
+ C
2
+ C
3
) 100(C
1
+ C
2
+ C
3
) - (100 - C
1
)(C
1
- C
3
) - (100 - C
2
)(C
2
- C
3
) =
= C
1
2
+ C
2
2
+ C
3
(300 - C
1
- C
2
) >
> C
1
2
+ C
2
2
+ C
3
(C
3
+ C
4
+ ... + C
100
)
C
1
2
+ C
2
2
+ C
3
2
+ ... + C
100
2
> 10000.
Suy ra, C
1
+ C
2
+ C
3
> 100.

Bi tp

1. Trong mi ca bng 2 x n ta vit cc s thc dng sao cho tng cc s ca mi ct bng 1. Chng
minh rng ta c th xo i mi ct mt s sao cho mi hng, tng ca cc s cn li khng vt qu
.
4
1 + n

2. 40 tn trm chia 4000 euro. Mt nhm gm 5 tn trm c gi l ngho nu tng s tin m chng
c chia khng qu 500 euro. Hi s nh nht cc nhm trm ngho trn tng s tt c cc nhm 5 tn
trm bng bao nhiu?

Nguyn l cc hn v phng trnh Diophant

Vietnamese IMO Team Training Camp 2010
38 | Trn Nam Dng 6/2010

Nguyn l cc hn ng dng trong phng trnh Diophant c nhc ti trong bi
phng php chng minh phn chng. Trong phn ny, ta trnh by chi tit ba v d p
dng nguyn l cc hn trong phng trnh Fermat, phng trnh Pell v phng trnh
dng Markov.

V d 1. Chng minh rng phng trnh x
4
+ y
4
= z
2
(1) khng c nghim nguyn dng.

Gi s ngc li, phng trnh (1) c nghim nguyn dng, v (x, y, z) l nghim ca
(1) vi z nh nht.

(1) D thy x
2
,y
2
,z i mt nguyn t cng nhau

(2) T nghim ca phng trnh Pythagore, ta c tn ti p, q sao cho
x
2
= 2pq
y
2
= p
2
- q
2

z = p
2
+ q
2


(3) T y, ta li c mt b ba Pythagore khc, v y
2
+ q
2
= p
2
.

(4) Nh vy, tn ti a,b sao cho
q = 2ab
y = a
2
- b
2

p = a
2
+ b
2

a,b nguyn t cng nhau

(5) Kt hp cc phng trnh ny, ta c:
x
2
= 2pq = 2(a
2
+ b
2
)(2ab) = 4(ab)(a
2
+ b
2
)

(6) V ab v a
2
+ b
2
nguyn t cng nhau, ta suy ra chng l cc s chnh phng.

(7) Nh vy a
2
+ b
2
= P
2
v a = u
2
, b = v
2
. Suy ra P
2
= u
4
+ v
4
.

(8) Nhng by gi ta thu c iu mu thun vi tnh nh nht ca z v:
P
2
= a
2
+ b
2
= p < p
2
+ q
2
= z < z
2
.

(9) Nh vy iu gi s ban u l sai, suy ra iu phi chng minh.

V d 2. Tm tt c cc cp a thc P(x), Q(x) tha mn phng trnh
P
2
(x) = (x
2
-1)Q
2
(x) + 1 (1)
Gii. Khng mt tnh tng qut, ta ch cn tm nghim trong tp cc a thc c h s khi
u dng.
Nu ) ( 1 ) ( ) 1 (
2 2
x Q x x P x x
n n
n
+ = + (2) th ) ( 1 ) ( ) 1 (
2 2
x Q x x P x x
n n
n
= (3)
Vietnamese IMO Team Training Camp 2010
39 | Trn Nam Dng 6/2010

Nhn (2) v (3) v theo v, ta c
) ( ) 1 ( ) (
)) ( 1 ) ( ))( ( 1 ) ( ( ) 1 ( ) 1 ( 1
2 2 2
2 2 2 2
x Q x x P
x Q x x P x Q x x P x x x x
n n
n n n n
n n
=
+ = + =

Suy ra cp a thc P
n
(x), Q
n
(x) xc nh bi (2) (v (3)!) l nghim ca (1). Ta chng
minh y l tt c cc nghim ca (1). Tht vy, gi s ngc li, tn ti cp a thc
P(x), Q(x) khng c dng P
n
(x), Q
n
(x) tha mn (1). Ta xt cp a thc (P, Q) nh vy
vi degQ nh nht.

t ) ( * 1 ) ( * ) 1 ))( ( 1 ) ( (
2 2 2
x Q x x P x x x Q x x P + = + (4)
Th r rng
) ( * 1 ) ( * ) 1 ))( ( 1 ) ( (
2 2 2
x Q x x P x x x Q x x P = +
Suy ra (P*, Q*) cng l nghim ca (1).
Khai trin (4), ta thu c P*(x) = xP(x) (x
2
-1)Q(x), Q*(x) = xQ(x) P(x). Ch l t
(1) ta suy ra (P(x) xQ(x))(P(x)+xQ(x)) = - Q
2
(x) + 1. V P(x) v Q(x) u c h s khi
u > 0 v degP = degQ + 1 nn ta c deg(P(x)+xQ(x)) = degQ + 1. T y, do deg(-
Q
2
(x) + 1) 2deg(Q) nn ta suy ra deg(Q*(x)) deg(Q) 1 < deg Q.

Nh vy, theo cch chn cp (P, Q) th tn ti n sao cho (P*, Q*) = (P
n
, Q
n
).

Nhng khi t (4) suy ra
1 2 2 2
2 2 2
) 1 ( ) 1 ( ) 1 (
) 1 ))( ( * 1 ) ( * ( ) ( 1 ) (
+
+ = + + =
+ + = +
n n
x x x x x x
x x x Q x x P x Q x x P

Suy ra (P, Q) = (P
n+1
,Q
n+1
), mu thun.
Vy iu gi s l sai v ta c iu phi chng minh.

V d 3. Tm tt c cc gi tr k sao cho phng trnh (x+y+z)
2
= kxyz c nghim nguyn
dng.

V d 4. (CRUX, Problem 1420) Nu a, b, c l cc s nguyn dng sao cho
0 < a
2
+ b
2
abc c
Chng minh rng a
2
+ b
2
abc l s chnh phng.

Gii. Gi s ngc li rng tn ti cc s nguyn dng a, b, c sao cho 0 < a
2
+ b
2
abc
c v k = a
2
+ b
2
abc (1) khng phi l s chnh phng.

By gi ta c nh k v c v xt tp hp tt c cc cp s nguyn dng (a, b) tha mn
phng trnh (1), tc l ta xt
S(c, k) = {(a, b) (N*)
2
: a
2
+ b
2
abc = k}
Gi s (a, b) l cp s thuc S(c, k) c a + b nh nht. Khng mt tnh tng qut c th
gi s a b. Ta xt phng trnh
Vietnamese IMO Team Training Camp 2010
40 | Trn Nam Dng 6/2010

x
2
bcx + b
2
k = 0
Ta bit rng x = a l mt nghim ca phng trnh. Gi a
1
l nghim cn li ca phng
trnh ny th a
1
= bc a = (b
2
k)/a.

Ta c th chng minh c rng (bn c t chng minh!) a
1
nguyn dng. Suy ra
(a
1
, b) cng thuc S(c, k).
Tip theo ta c a
1
= (b
2
-k)/a < a
2
/a = a, suy ra a
1
+ b < a + b. iu ny mu thun vi cch
chn (a, b).

Bi tp

1. (IMO 88) Nu a, b, q = (a
2
+b
2
)/(ab+1) l cc s nguyn dng th q l s chnh phng.

2. (PTNK 03). Tm tt c cc s nguyn dng k sao cho phng trnh x
2
- (k
2
-4)y
2
= - 24 c nghim
nguyn dng.

3. (Mathlinks) Cho A l tp hp hu hn cc s nguyn dng. Chng minh rng tn ti tp hp hu hn
cc s nguyn dng B sao cho A B v
xB
x =
xB
x
2
.

4. (AMM 1995) Cho x, y l cc s nguyn dng sao cho xy + x v xy + y l cc s chnh phng.
Chng minh rng c ng mt trong hai s x, y l s chnh phng.

5. (IMO 2007) Cho a, b l cc s nguyn dng sao cho 4ab 1 chia ht (4a
2
-1)
2
. Chng minh rng a = b.

6. Sp th t

Sp th t l mt cch gim s trng hp cn xt v to ra cc iu kin b sung
gip chng ta c th lm vic d dng hn.

V d 1. Cho x
1
, x
2
, , x
n
l n s thuc on [0, 2]. Chng minh rng

= =

n
i
n
j
j i
n x x
1
2
1
. | |

V d 2. (Bt ng thc Schur m rng) Cho x, y, z l cc s thc khng m, r l mt s
thc dng bt k. Chng minh rng ta c bt ng thc
a
r
(a-b)(a-c) + b
r
(b-a)(b-c) + c
r
(c-a)(c-b) 0

V d 3. Cho a, b, c l cc s thc i mt khc nhau. Chng minh rng

( )
2
9
) (
1
) (
1
) (
1
2 2 2
2 2 2

|
|

\
|

+ +
a c c b b a
c b a


Bi tp

1. Cho x, y, z thuc [0, 1]. Chng minh rng (x+y+z)((x-y)
2
+(y-z)
2
+(z-x)
2
) 4.
Vietnamese IMO Team Training Camp 2010
41 | Trn Nam Dng 6/2010


2. (IMO 2006) Tm s thc M nh nht sao cho bt ng thc
|ab(a
2
- b
2
) + bc(b
2
- c
2
) + ca(c
2
- a
2
)| < M(a
2
+ b
2
+ c
2
)
2

ng vi mi s thc a, b, c.

3. (Vietnam MO 2008) Cho x, y, z l cc s thc khng m khc nhau. Chng minh rng
. 4
) (
1
) (
1
) (
1
) (
2 2 2

(

+ +
x z z y y x
zx yz xy

Ti liu tham kho

1. on Qunh, Don Minh Cng, Trn Nam Dng, ng Hng Thng, Ti liu gio
khoa chuyn ton, i s 10, Nh xut bn Gio dc 2009.

2. on Qunh, Don Minh Cng, Trn Nam Dng, ng Hng Thng, Ti liu gio
khoa chuyn ton, Bi tp i s 10, Nh xut bn Gio dc 2009.

3. Nguyn Vn Mu, Trn Nam Dng, V nh Ha, ng Huy Run, Chuyn chn
lc T hp v Ton ri rc, Nh xut bn Gio dc 2008.

4. Arthur Engel, Problem Solving Strategies, Springer 1998.

5. N.Agakhanov, Cc bi thi Olimpic Ton ton Nga 1996-2006, Nh xut bn MCCME
2007.

6. A. Kovaldzi v A. Kanel-Belov, Gii bi ton khng mu mc nh th no, Nh xut
bn MCCME 2006.

7. Stasys Jukna, Extremal Combinatorics, Springer 2001.














Vietnamese IMO Team Training Camp 2010
42 | Trn Nam Dng 6/2010


Nguyn l chung v th

Nguyn l chung v th (hay cn c gi l nguyn l Dirichlet) khng nh mt s
kin hin nhin rng n+1 con th khng th c xp vo n chung sao cho mi con
th u ring mt chung. Mt cch tng qut hn, nguyn l chung v th khng
nh rng:

Nu mt tp hp gm nhiu hn kn i tng c chia thnh n nhm, th c mt nhm
no c nhiu hn k i tng.

Chn l ny rt d kim tra: nu nhm no cng c nhiu nht k i tng th tng cng
ch c nhiu nht kn i tng c chia ra.

y l mt trong nhng nguyn l khng xy dng (non-constructive) lu i nht: n
ch ni n s tn ti ca mt chung trong c nhiu hn k vt m khng ni g n
cch tm ra chung ny. Ngy nay chng ta c nhng tng qut ha rt mnh ca
nguyn l ny (cc nh l kiu Ramsey, phng php xc sut).

Mc d nguyn l chung v th c pht biu rt n gin, n c hng lot cc ng
dng khng tm thng. Ci kh ca vic ng dng nguyn l ny l xc nh c xem
th l g v chung l g. Chng ta s minh ha iu ny bng mt s v d.

1. Mt s v d m u

khi ng, chng ta s bt u bng nhng ng dng n gin nht. Bc ca mt nh
trong th G l s d(x) cc cnh ca G k vi x.

Mnh 1. Trong mi th tn ti hai nh c cng bc.

Chng minh. Gi s ta c th G c n nh. Ta to ra n ci chung c nh s t 0
n n-1 v xp nh x vo chung th k khi v ch khi d(x) = k. Nu nh trong mt
chung no c nhiu hn 1 nh th ta c pcm. V th ta c th gi s rng khng c
chung no cha hn 1 nh. C tt c n nh c chia vo n ci chung, nhng vy mi
mt chung c ng 1 nh. Gi x v y l cc nh nm trong cc chung nh s 0 v n-
1 tng ng. nh x c bc 0 v vy n khng c ni vi cc nh khc, trong c y.
Nhng y c bc n-1 nn n li c ni vi tt c cc nh, trong c x, mu thun.

Nu G l mt th hu hn, ch s c lp (independent number) (G) l s ln nht
cc nh i mt khng k nhau ca G. Sc s (chromatic number) (G) ca G l s nh
nht cc mu cn dng t cc mu ca G sao cho khng c hai nh k nhau c t
cng mu.
Vietnamese IMO Team Training Camp 2010
43 | Trn Nam Dng 6/2010


Mnh 2. Trong mi th G vi n nh ta c n (G).(G).

Chng minh. Ta chia cc nh ca G thnh (G) nhm (cc tp hp cc nh c cng
mu). Theo nguyn l chung v th, mt trong cc nhm c cha t nht n/(G) nh,
v cc nh ny i mt khng k nhau. Nh vy (G) n/(G) v chnh l iu cn
chng minh.

Mt th l lin thng nu gia hai nh bt k ca n c mt ng i.

Mnh 3. Cho G l mt th n nh. Nu mi nh ca G c bc t nht l (n-1)/2
th G lin thng.

Chng minh. Ta xt hai nh x, y bt k. Nu hai nh ny khng k nhau th c t nht n-
1 nh ni chng vi cc nh cn li, v c x v y u c bc t nht l (n-1)/2. V ch cn
n-2 nh khc, nguyn l chung v th suy ra rng phi c mt trong cc nh ni vi
c x v y. Ta chng minh c rng mi cp nh th hoc k nhau, hoc c nh k
chung, v nh vy G lin thng.

Ghi ch. Mt kt qu l tt nht nu nh kt lun khng cn ng khi ta lm yu i mt
iu kin. V d, trong kt qu trn: gi s n l chn v G l hp ca hai th y
vi n/2 nh th bc ca mi nh bng (n-2)/2 nhng th khng lin thng.

Bi tp 1. Gi s 5 im c chn trong hnh vung cnh 1. Chng minh rng tn ti t nht 1 cp im
cch nhau khng qu 1/2.

Bi tp 2. Cc vin ca 8 mu khc nhau c xp vo 6 ci hp. C 20 vin cho mi mu. Chng
minh rng tm c mt hp cha hai cp c cng mu khc nhau.

Bi tp 3. Chng minh rng mt tp hp bt k gm n+1 phn t c chn t {1, 2,,2n} u cha
mt cp phn t c tng bng 2n+1. Hy chng minh kt qu ny l tt nht.

Bi tp 4. Chng minh rng mt tp hp bt k gm n+1 s nguyn c chn t {1, 2,, 2n} cha hai
s m s ny chia ht cho s kia.

2. nh l Erdos-Szekeres

Cho A = (a
1
, a
2
,, a
n
) l dy gm n s phn bit. Mt dy con k phn t ca A l dy B
gm k s hng phn t ca A xut hin theo ng th t m chng xut hin trong A. C
ngha l B = (a
i1
, a
i2
,, a
ik
) vi i
1
< i
2
< < i
k
. Dy con B c gi l tng nu a
i1
< a
i2
<< a
ik
, v gim nu a
i1
> a
i2
>> a
ik
.
Ta quan tm n di ln nht ca dy con tng v gim ca A. Suy lun trc quan cho
thy phi c mt s cn i nht nh gia hai di ny. Nu nh dy con tng di nht
l ngn, chng hn c chiu di l s, th mi dy con ca A c di s+1 phi cha cp
Vietnamese IMO Team Training Camp 2010
44 | Trn Nam Dng 6/2010

phn t gim, nh vy c rt nhiu cp phn t gim. V th ta trng i rng dy con
gim di nht s ln. Mt trng hp cc bin xy ra khi s = 1. Khi c dy s A l
gim.

Lm sao ta c th s ha iu d cm rng di ca dy con tng di nht v dy con
gim di nht khng th cng nh ? Kt qu ni ting ca Erdos v Szekeres (1935) cho
chng ta cu tr li cho cu hi ny v y l mt trong nhng kt qu u tin ca ti u
t hp.

nh l 4 (Erdos-Szekeres 1935). Cho A = (a
1
, a
2
,, a
n
) l dy gm n s thc phn bit.
Nu n rs + 1 th hoc A c dy con tng di s+1 hoc A c dy con gim di r+1
(hay c hai).

Chng minh. (ca Seidenberg 1959). Ta cho tng ng mi phn t a
i
ca A vi cp
im s (x
i
, y
i
) trong x
i
l s phn t ca dy con tng di nht kt thc ti a
i
v y
i
l
s phn t ca dy con gim di nht bt u t a
i
. Ch rng khng c hai phn t no
c cng im s, tc l (x
i
, y
i
) (x
j
, y
j
) vi mi i j. Tht vy, nu ta c ... a
i
... a
j
..., th
hoc a
i
< a
j
v dy con tng di nht kt thc ti a
i
c th ko di n a
j
(v do x
i
< x
j
),
hoc a
i
> a
j

v dy con gim di nht bt u t a
j
c th c bt u t a
i
(v nh th y
i

> y
j
).

By gi ta to ra mt li gm n chung th.

n


s



1









1 r n

Ta t mi phn t a
i

vo chung vi ta (x
i
, y
i
). Mi mt phn t ca A c th c
t vo mt chung v 1 x
i
, y
i
n vi mi i = 1, 2, ..., n. Hn na, khng c chung
no c cha nhiu hn mt phn t, v (x
i
, y
i
) (x
j
, y
j
) vi mi i j. V |A| = n rs +
1, ta c nhiu vt hn l s chung th c t m trong hnh v trn. Nh vy phi c
mt phn t a
i
nm ngoi min t m. Nhng iu ny c ngha l x
i
s+1 hoc y
i
r +
1 (hoc c hai), ng iu chng ta cn.
Tp hp cc s thc c sp ton phn. iu ny c ngha l vi hai s phn bit x, y
th hoc x < y hoc y < x. B di y, thuc v Dilworth, s tng qut ha nh l
Erdos-Szekeres cho cc tp hp m trong hai phn t c th khng so snh c.
Vietnamese IMO Team Training Camp 2010
45 | Trn Nam Dng 6/2010


Mt th t b phn (yu) trn tp hp P l quan h hai ngi < gia cc phn t ca P. Ta
ni hai phn t x v y l so snh c nu x < y hoc y < x (hoc c hai). Mt xch l
mt tp hp Y P sao cho hai phn t bt k ca Y l so snh c. Nu khng c hai
phn t khc nhau no ca Y l so snh c, th Y c gi l i xch.

B 5 (Dilworth 1950). Trong mi th t b phn trn tp hp P gm n sr + 1 phn
t, tn ti xch c kch thc s+1 hoc i xch c kch thc r+1.

Chng minh. Gi s rng khng c xc di s+1. Khi ta c th nh ngha hm s f:
P {1,..., s} trong f(x) l s phn t ln nht ca mt xch c phn t ln nht x.
Theo nguyn l chung v th, s c r+1 phn t ca P c cng nh qua nh x f. Theo
nh ngha ca f, cc phn t ny khng so snh c; v nh vy chng to thnh mt
i xch c kch thc r+1.

Bi tp 5. T b Dilworh hy suy ra nh l Erdos-Szekeres.

Bi tp 6. Cho n
2
+1 im trong mt phng. Chng minh rng tn ti dy gm n+1 im
(x
1
,y
1
),(x
2
,y
2
),,(x
n+1
,y
n+1
) sao cho x
1
x
2
x
n+1
v y
1
y
2
y
n+1
, hoc dy gm n+1 im sao
cho x
1
x
2
x
n+1
v y
1
y
2
y
n+1


3. nh l Mantel

Di y chng ta s tho lun v mt tnh cht ti u c trng ca th. Mt th G
gm 2n nh khng cha tam gic c th c bao nhiu cnh? Tam gic l tp hp {x, y,
z} gm ba nh m hai nh bt k u c ni vi nhau bi mt cnh. D nhin l G c
th cha n
2
cnh m khng cha tam gic: ch cn ly th hai phe y gm hai tp
hp mi tp hp c n nh v tt c cc cnh ni gia hai tp hp. Thc t l n
2
chnh l
s cnh ln nht c th: nu ta thm mt cnh v th th s xut hin tam gic.

Ta s a ra 4 chng minh cho kt qu p ny: chng minh th nht dng nguyn l
chung v th, chng minh th hai da trn phng php m bng hai cch, chng
minh th ba s dng bt ng thc AM-GM v chng minh th t s dng l lun dch
chuyn (ta s cp ti chng minh ny trong phn sau).

nh l 6 (Mantel 1907). Nu th G vi 2n nh c n
2
+1 cnh th G cha tam gic.

Chng minh th nht. Ta chng minh bng quy np theo n. Vi n = 1, th G khng th c
n
2
+1 cnh v v vy mnh ng. Gi s mnh ng n n, ta xt th G vi
2(n+1) nh v (n+1)
2
+ 1 cnh. Gi x v y l hai nh k nhau trong G, v H l th con
cm sinh trn 2n nh cn li. Nu H cha t nht n
2
+1 cnh th theo gi thit quy np, ta
c ngay pcm. Gi s rng H c nhiu nht n
2
cnh, khi c t nht 2n+1 cnh ca G s
ni t x v y n cc nh ca H.

Vietnamese IMO Team Training Camp 2010
46 | Trn Nam Dng 6/2010

Theo nguyn l chung v th, gia 2n+1 cnh ny c t nht 1 cnh ni t x v mt cnh
ni t y n cng mt nh z thuc H. Nh vy G cha tam gic {x, y, z}.

Chng minh th hai. Cho G l th trn tp hp V gm 2n nh v c m n
2
+1 cnh.
Gi s rng G khng cha tam gic. Khi cc cnh k nhau khng c nh k chung, do
d(x) + d(y) 2n vi mi cnh {x, y} E. Cng theo tt c cc cnh ca G, ta c
. 2 )) ( ) ( ( ) (
} , {
2
mn y d x d x d
E y x V x
+ =



Mt khc, s dng bt ng thc Cauchy-Schwarz v ng thc Euler m x d
V x
2 ) ( =

, ta
c

=
|

\
|
=
V x
V x
n
m
V
x d
x d .
2
| |
) (
) (
2
2
2

T hai bt ng thc ny suy ra m n
2
, mu thun vi gi thit.

Chng minh th ba. Gi s G = (V, E) l th trn tp V gm 2n nh v gi s G
khng cha tam gic. Gi s A V l tp hp c lp ln nht, tc l tp hp ln nht
cc nh sao cho khng c nh no k nhau trong G. V G khng cha tam gic tt c
cc nh k vi nh x V to thnh mt tp c lp v ta suy ra d(x) |A| vi mi x.

Tp hp B = V A giao vi mi cnh ca G. Tnh cc cnh ca G tng ng vi nh
cui ca chng trong B, ta c

B x
x d E ) ( | | . By gi bt ng thc AM-GM cho ta
.
2
| | | |
| | . | | ) ( | |
2
2
n
B A
B A x d E
B x
= |

\
| +





4. nh l Turan

Mt k-clique l mt th vi k nh m hai nh bt k u c ni vi nhau bi mt
cnh. V d tam gic l 3-clique. nh l Mantel khng nh rng nu th vi n nh
khng cha 3-clique th n c nhiu nht n
2
/4 cnh. Cn nu k > 3 th sao?

Cu tr li c cho bi kt qu c bn ca Paul Turan, kt qu m u cho l thuyt
th ti u.

nh l 7 (Turan 1941). Nu th G = (V, E) trn n nh khng cha (k+1)-clique, k
2, th
.
2
1
1 | |
2
n
k
E |

\
|
(1)
Cng ging nh nh l Mantel, nh l ny c nghin cu nhiu ln vi nhiu cc
chng minh khc nhau. y chng ta a ra chng minh nguyn thy ca Turan. Php
Vietnamese IMO Team Training Camp 2010
47 | Trn Nam Dng 6/2010

chng minh da trn l lun dch chuyn trng lng s c cp trong phn bi
tp. Ngoi ra s cn mt cch chng minh s dng mt tng hon ton khc l lun
xc sut.

Chng minh. Ta s dng php quy np theo n. Khi n = 2, bt ng thc (1) l hin nhin
ng. Trng hp k=2 chnh l nh l Mantel. By gi gi s bt ng thc ng cho
mi th trn nhiu nht n-1 nh, v G = (V, E) l th trn n nh khng c (k+1)-
clique v c s cnh ln nht. th ny d nhin l phi cha k-clique, bi nu khng ta
c th thm cnh. Gi s A l k-clique v B = V A.

V mi cp nh ca A c ni bi mt cnh, A cha
|
|

\
|
=
2
k
e
A
cnh. Gi e
B
l s cnh
ni cc nh ca B v e
A,B
l s cnh ni gia cc cnh ca A v B. Theo gi thit quy
np, ta c
.
2
) ( 1
1
2
k n
k
e
B

\
|

V G khng c k+1 clique nn mi x B k vi nhiu nht k-1 nh thuc A, v ta thu
c
e
A,B
(k-1)(n-k).
Cng cc bt ng thc ny li v s dng ng thc

2
2
2 2 2
1
1 |

\
|
|
|

\
|
= |

\
|

n
k
n
k

ta suy ra rng

.
2
1
1 1
2
) )( 1 (
2 2 2
| |
2
2
2
,
n
k k
k n
k
k n k
k n
k k
e e e E
B A B A
|

\
|
= |

\
|

|
|

\
|
=
+ |

\
|
|
|

\
|
+
|
|

\
|
+ +


Bi tp 7. Gi s rng n l bi s ca k. Hy xy dng mt th khng cha (k+1)-clique, trong s
cc cnh t c cn trn (1) trong nh l 7.

Bi tp 8. Nhc li ch s c lp (G) ca th G l s ln nht cc nh i mt khng k nhau ca G.
Hy chng minh i ngu ca nh l Turan: Nu G l th vi n nh v nk/2 cnh, k 1, th (G)
n/(k+1).

5. nh l Dirichlet

y ta trnh by mt ng dng ca nguyn l chung v th m Dirichlet s dng,
v chnh v ng dng ny m nguyn l ny c gn vi tn ng. N lin quan n vn
tn ti xp x hu t tt cho cc s v t. Kt qu ny thuc v l thuyt s nhng l
lun l t hp.
Vietnamese IMO Team Training Camp 2010
48 | Trn Nam Dng 6/2010


nh l 8 (Dirichlet 1879). Nu x l mt s thc. Vi mi s nguyn dng n,tn ti s
hu t p/q sao cho 1 q n v
.
1 1
2
q nq q
p
x <
Chng minh. Cho chng minh ny, ta gi {x} l phn l ca s thc x, tc l {x} = x
[x].

Nu x l s hu t th khng c g chng minh. V th, ta gi s rng x l v t v xt
n+1 s {ax}, a = 1, 2, , n+1. Ta t n+1 s ny vo n chung
. 1 ,
1
,...,
2
,
1
,
1
, 0 |

\
|
|

\
|
|

\
|
n
n
n n n

(Khng c s no trong cc s ni trn trng vi u mt cc on, v x l v t). Theo
nguyn l chung v th, c mt on no cha nhiu hn mt s, v d l {ax} v
{bx}vi a > b, v do cch nhau khng qu 1/n. t q = a b, ta thy rng tn ti s
nguyn p sao cho |qx p| < 1/n, t suy ra kt qu cn chng minh bng cch chia cho
q. Hn na, q l hiu ca hai s nguyn thuc 1, 2, , n+1, do q n.

6. th c t c sc

Ta t mu cc cnh ca th y K
n
trn n nh. Ta ni rng th c t c sc
(swell-colored) nu mi tam gic cha 1 hoc 3 mu, nhng khng cha 2 mu v th
c nhiu hn mt mu. C ngha l, ta cn s dng t nht 2 mu v vi mi tam gic,
hoc l tt c cc cnh ca n c cng mu hoc c mu khc nhau.

Ta c th chng minh c rng (hy chng minh!) K
n
khng th c t c sc vi
ng hai mu. Cng c th thy rng K
3
v K
4
l nhng th K
n
duy nht c th t c
sc vi 3 mu; cc th K
n
khc cn nhiu mu hn v chng c bc lin thng cao hn.

S dng nguyn l chung v th, ta c th chng minh c chn di sau.

nh l 9 (Ward-Szabo 1994). th y trn n nh khng th c t c sc vi t
hn 1 + n mu.

Chng minh. Gi s K
n
c t c sc vi r mu khc nhau. Gi N(x, c) l s cnh k
vi nh x c t mu c. C nh x
0
v c
0
sao cho N(x
0
, c
0
) ln nht, v k hiu gi tr
ln nht ny l N.

n-1 cnh k vi x
0
c th c chia thnh r nhm mu, mi nhm c N hoc t hn
phn t. Theo nguyn l chung v th
N.r n 1.
Vietnamese IMO Team Training Camp 2010
49 | Trn Nam Dng 6/2010

Gi x
1
, x
2
, , x
N
l cc nh k vi x
0
bi N cnh c mu c
0
. Gi G l th con (y )
ca K
n
cm sinh t tp hp cc nh {x
0
, x
1
,..., x
N
}. Tnh c sc ca K
n
c cm sinh
cho G v nh vy mi cnh ca G c cng mu c
0
. V K
n
c t nht hai mu nn tn ti
nh y thuc K
n
khng nm trong G v sao cho t nht mt cnh ni y vi G c mu khc
c
0
.

Khng nh 10. N+1 cnh ni y ti G c t mu khc nhau v khc vi c
0
.

T khng nh ny ta suy ra r N + 2, t , cng vi bt ng thc N.r n 1 suy ra
r(r-2) n 1, v t 1 + n r l iu cn chng minh. Nh vy ta ch cn cn chng
minh khng nh trn.

Nu mt cnh ni y ti G, v d {y, x
1
}, c mu c
0
th theo tnh c sc, cnh {y,x
0
} phi
c t mu c
0
, mu thun vi cch chn y (nhc li l x
1
, x
2
,,x
N
l tt c cc nh k
vi x
0
c ni bng cnh mu c
0
). Tip theo, nu hai cnh no ni y vi G, chng
hn {y, x
1
} v {y, x
2
} c cng mu th theo tnh c sc ca K
n
ta c cnh {x
1
, x
2
} cng
c t bng mu . Nhng {x
1
, x
2
} thuc G v c mu c
0
v nh vy {y, x
1
} phi c
mu c
0
l iu m ta chng minh trn l khng th. iu ny kt thc php chng
minh khng nh v cng l kt thc chng minh nh l.

Tnh ti u ca cn di cho bi nh l 9 c th c chng t s dng mt cu hnh
gi l mt phng afine. Ta hiu rng mt phng afine AG(2,q) bc q cha ng q
2
im
v ng q+1 lp (c gi l bt ch) cc ng thng song song, mi lp cha q
ng thng (hai ng thng l song song nu chng khng c im chung). Hn na,
hai im bt k nm trn ng mt ng thng.

Khi c 1 mt phng nh vy, ta c th xy dng mt php t c sc cho
2
q
K vi q+1
mu nh sau. Ta ng nht cc nh ca
2
q
K vi cc im ca AG(2,q) v cho tng ng
mt mu duy nht cho mi mt trong s q+1 bt ch cc ng thng song song. xc
nh cch t c sc, ta xt hai im khc nhau x v y ca
2
q
K . Hai im ny thuc duy
nht mt ng thng v ng thng ny, n lt mnh thuc duy nht mt bt ch. Ta
t mu cnh {x,y} bng mu ca bt ch ny. V hai im bt k nm trn mt ng
thng duy nht v hai ng thng song song khng c im chung, mi cnh ca mt
tam gic s c t bng cc mu khc v nh vy, php t l c sc nh mong mun.

Thc t, Ward v Szabo (1994) chng minh rng iu ngc li cng ng: nu th
2
q
K (q 2) c th t c sc bng q+1 mu th php t ny c th c dng xy dng
mt phng afine bc q.

[Trch t cun Extremal Combinatorics ca Stasys Jukna]

Vietnamese IMO Team Training Camp 2010
50 | Trn Nam Dng 6/2010


Gii phng trnh hm bng cch lp phng trnh

Trn Nam Dng
Trng H KHTN Tp.HCM

Cuc sng l chui nhng phng trnh
m ta kim tm li gii

Gii bi ton bng cch lp phng trnh v h phng trnh l mt phng php thng
dng trong cc bi ton i s. tng l tm mt n s no , ta a vo cc n s
ph, s dng cc d kin cho to ra mi lin h gia cc n s (cc phng trnh),
gii h phng trnh, tm ra gi tr ca n s cn tm. Phng php tng t cng c th
p dng cho cc bi ton hnh hc tnh ton (chng hn bi ton gii tam gic, t gic),
cc bi ton m (phng php dy s ph).

Trong bi ny, chng ta cp ti phng php lp phng trnh, h phng trnh gii
cc bi ton phng trnh hm. tng chung cng l tm mt gi tr f(x) hoc f(a)
no , ta s dng phng trnh hm tm ra mi lin kt gia cc i lng, ni cch
khc, to ra cc phng trnh s. Gii cc phng trnh s ny, ta c th tm ra f(x) hoc
f(a) vi a l mt gi tr no .

1. Gii phng trnh hm da vo tnh xon ca hm s

Hm s (x) c gi l xon i vi php hp nu tn ti s nguyn dng k sao cho

k
(x) = x vi mi x. y
1
(x)=(x),
2
(x)=((x)), ,
k
(x) = (
k-1
(x)). Ta c th
s dng tnh xon ca mt s hm s gii mt s dng phng trnh hm vi mt bin
t do. Chng ta bt u t nhng v d n gin:

Bi ton 1. Tm tt c cc hm s f: R \ {0} R tho mn iu kin
( )
2
1
) 1 ( x
x
f x f x =
|

\
|
+ (1) vi mi x 0
Gi
i. Trong (1), thay x bng 1/x, ta c
( )
2
1 1
1
1
x
x f
x
f
x
= +
|

\
|
|

\
|
(2)
Gii h (1), (2), coi f(x) v f(1/x) l cc n s, ta c

x x x
x x
x f
+
+
=
2 3
3 4
1
) ( .

Vietnamese IMO Team Training Camp 2010
51 | Trn Nam Dng 6/2010

Trong v d trn, ta cn tm f(x). Ta coi f(x) l n s chnh, cn f(1/x) l mt n ph.
Phng trnh (1) cho chng ta mt s lin kt gia n s chnh v n ph. Thay x bng
1/x, ta hy vng s tm ra c mi lin kt mi. Trong trng hp ny, chnh l
phng trnh (2). V phng trnh (2) khng to ra n s mi, tc l s phng trnh
bng s n s, cho gii tm c f(x) nn ta dng li. Trong v d tip theo, chng ta
s thy qu trnh to cc mi lin kt c th to ra cc n s mi.

Bi ton 2. Tm tt c cc hm s f: R \ {0, 1} R tho mn iu kin

1
1
1
1
) 1 ( ) (
2

+
=
|

\
|

+
+ +
x
x
x
x
f x x f (1)
Gii. n s y ang l f(x) v n s ph l
|

\
|

+
x
x
f
1
1
. (1) cho ta mt phng trnh kt
ni 2 n s ny. Ta tm cc mi lin kt khc. Trong (1) ta thay x bng
x
x

+
1
1
th c
) 2 (
) 1 (
1 1
1
2
1
1
1
1
1
1
1
1
1
1
1
1
1
1
1
1
1
1
1
2
2

+
= |

\
|

+ |

\
|

+
+ |

\
|

+
=
|
|
|
|

\
|

+
+
|

\
|
+

+
+
|

\
|

+
x x
x
x
f
x x
x
f
x
x
x
x
x
x
x
x
f
x
x
x
x
f

Nh vy, trong (2) li xut hin mt n s ph mi, l .
1
|

\
|

x
f Nu dng li y th
s n s nhiu hn s phng trnh v ta s khng tm c f(x). Trong (1), tip tc thay
x bng -1/x th ta c

) 3 (
) 1 (
1
1
1
1
1 1
1
1
1
1
1
1
1
1
1
1 1
2
2
+
+
= |

\
|
+

\
|
+ + |

\
|


+
=
|
|
|
|

\
|
+

\
|
+ + |

\
|

x x
x
x
x
f
x x
f
x
x
x
x
f
x x
f Ta
tip tc nhn c mt n s ph mi, l .
1
1
|

\
|
+

x
x
f . Trong (1), ta li thay x bng
x
x
+

1
1
th c
1
1
1
1
1
1
1
1
1
1
1
1
1
1
1
1
1
2

+ |

\
|
+

=
|
|
|
|

\
|
+

+
+

\
|
+
+

+ |

\
|
+

x
x
x
x
x
x
x
x
f
x
x
x
x
f
( ) ) 4 (
1
1
1
2
1
1
2
+
+
=
+

\
|
+


x
x
x f
x
x
x
x
f
Vietnamese IMO Team Training Camp 2010
52 | Trn Nam Dng 6/2010

n y th n s mi khng xut hin, nh vy ta c 4 phng trnh (1), (2), (3),
(4) vi 4 n s l
|

\
|
+

\
|

\
|

+
x
x
f
x
f
x
x
f x f
1
1
,
1
,
1
1
), ( . Gii h ny nh mt h phng trnh
bc nht 4 n, ta tm c

) 1 (
1
) (
2

+
=
x x
x
x f .
Quan st li gii ca hai bi ton trn, ta thy rng phng php tng t c th p dng
cho bi ton tm tt c cc hm f(x) tho mn phng trnh
f(x) + a(x)f((x)) = b(x) (1)
Trong a(x), b(x) l cc hm s cho, cn (x) l mt hm cho tho mn tnh cht

k
(x) = x vi mi x. S k nh nht tho mn iu kin
k
(x) = x c gi l bc ca hm
s (x).

Cc v d thng gp v hm (x) chnh l (x) = -x, (x) = 1/x. Hm
x
x
x

+
=
1
1
) (
trong v d 2 l mt hm bc 4. C th rng kim tra hm s
x
x
x
3 1
3
) (

+
= c bc l 3.

Ch rng a s cc hm s l khng xon. Chng hn trong cc hm a thc, ch c
hm s f(x) = x v f(x) = a x l xon. Mt lp hm phn thc xon c m t phn
bi tp.

Ngay c vi phng trnh hm cha cc hm xon, ta cng c th gp kh khn khi do
tnh i xng, ta khng to ra s phng trnh gii (c th xy ra trng hp c
mt phng trnh no l h qu ca cc phng trnh khc). V d:

Bi ton 3. Tm tt c cc hm s f: R R tho mn iu kin
f(x) + f(-x) = x
2
vi mi x thuc R.

Gii. Trong ng thc f(x) + f(-x) = x
2
(1) thay x bng x th ta c
f(-x) + f(x) = (-x)
2
(2)
Nh vy khng xut hin n s mi nhng cng lc , (2) cng khng phi l phng
trnh mi, m hon ton ging nh (1). Nh vy, ta khng th gii c h (1), (2) ra
gi tr duy nht ca f(x).

Cng tng t nh trong trng hp h phng trnh tuyn tnh vi s n s nhiu hn
s phng trnh trng hp ny h ca chng ta, hay ni cch khc, phng trnh (1) s
c v s nghim. C th trong bi ton ny, t g(x) = f(x) x
2
/2 th ta c g(x) + g(-x)
= 0, suy ra g(x) l mt hm s l. Ngc li, nu f(x) = x
2
/2 + g(x) vi g(x) l mt hm s
l bt k th r rng ta c
f(x) + f(-x) = x
2
/2 + g(x) + x
2
/2 + g(-x) = x
2
.
Vietnamese IMO Team Training Camp 2010
53 | Trn Nam Dng 6/2010


Vy tt c cc hm s f(x) tho mn iu kin bi l f(x) = x
2
/2 + g(x), trong g(x) l
mt hm s l bt k xc nh trn R.

Bi tp

1. Tm tt c cc hm s f: R R sao cho vi mi x {-1, 1} ta c
x
x
x
f
x
x
f =
|

\
|

+
+
|

\
|
+

1
3
1
3


2. Gi s a 0. Tm hm s f(x) bit rng
x
x a
a
f x f =
|
|

\
|

+
2
) (

3. Tm hm s f(x) bit rng

2
3 2
4 13
1 2
2 3
3
1
2
x x
x
x
x
f
x
x
f

= |

\
|
+

\
|



4. a) Chng minh rng hm s
x
x
x

+
=
3
3 1
) ( l xon
b) Tm tt c cc gi tr a sao cho hm s
ax
x a
x

+
=
1
) ( l xon.

5. Tm tt c cc hm s f(x): R R sao cho vi mi x khc 0 ta c
f(x) + f(1/x) = 2

6. Tm tt c cc hm s f: R \ {0, 1} R tho mn phng trnh

) 1 (
) 2 1 ( 2
1
1
) (
x x
x
x
f x f

=
|

\
|

+
vi mi x thuc min xc nh ca f.

2. Gii phng trnh hm bng cch lp phng trnh

Vi nhng phng trnh hm c 2 (hoc nhiu hn) phng trnh iu kin, ta c th tm
cch kt hp cc phng trnh tm ra f(x). Phng php c bn vn l to ra cc
mi lin kt, hay cc phng trnh bng cch tnh mt gi tr bng hai cch khc nhau.

Bi ton 4. Tm tt c cc hm s f: R R tho mn iu kin
i) f(-x) = -f(x) vi mi x thuc R;
ii) f(x+1) = f(x) + 1 vi mi x thuc R;
Vietnamese IMO Team Training Camp 2010
54 | Trn Nam Dng 6/2010

iii)
2
) ( 1
x
x f
x
f = |

\
|
vi mi x khc 0.
Gii. Tt c cc iu kin u trn mt bin x. Trong trng hp ny, ta c th dng mt
cht khi nim v th hiu con ng i n li gii. Ta xem cc s thc nh cc
nh ca mt th. nh x s c ni vi cc nh x+1, -x, 1/x. Cc iu kin bi s
cho chng ta cc mi lin h gia gi tr ca hm s ti cc nh c ni bi mt cnh.
Nu chng ta tm c mt chu trnh th mt cch t nhin, chng ta s c 1 phng trnh
( trnh hm s c hai gi tr khc nhau).

Ta th tm mt chu trnh nh vy
x
x x x
x
x
x
x x x
x x + =
+

+
=
+

+

+
+
1 1
1
1
1 1
1
1
1
1
1
1
1
t y = f(x) th t chu trnh trn, ta ln lt c
2 2 2
) 1 (
1
1
1
,
) 1 (
1
1
1
,
) 1 (
1
1
1
, 1 ) 1 (
+
+
= |

\
|
+ +
+
= |

\
|
+

+
+
= |

\
|
+
+ = +
x
y
x
x
f
x
y
x
f
x
y
x
f y x f
y x x f
x
y x
x
f
x
y x x
x
x
x
y
x
x
f =

= |

\
| +
=
|

\
|
+
+
+

= |

\
| +
2 ) ( ,
2 1
,
2
1
) 1 (
1
1
1
2 2
2
2
2


T suy ra 2x y = y, tc l y = x. Vy f(x) = x.

Trong l lun trn, ta cn n iu kin x khc 0 v -1. Tuy nhin t iu kin f(x+1) =
f(x) + 1 ta d dng suy ra f(0) = 0 v f(-1) = 1. Vy f(x) = x l tt cc nghim ca bi
ton.

Bi ton 5. Tm tt c cc hm s f: R R tho mn iu kin
f(x
2
-y) = xf(x) f(y) vi mi x, y thuc R
Gii. Thay x = y = 0 vo phng trnh hm, ta c f(0) = - f(0), suy ra f(0) = 0. Thay y
= 0 v phng trnh hm, ta c
f(x
2
) = xf(x) (1)
T suy ra
f(x
2
y) = f(x
2
) f(y)
Thay x = 0, ta c f(y) = f(y). Thay y bng y, ta c
f(x
2
+y) = f(x
2
) f(y) = f(x
2
) + f(y) vi mi x, y.
T , kt hp vi tnh cht hm l, ta suy ra f(x+y) = f(x) + f(y) vi mi x, y.
By gi ta c f((x+1)
2
) mt mt c th tnh theo cng thc (1), tc l bng (x+

1)f(x+1) = (x+1)(f(x)+f(1)). Mt khc, ta c th khai trin
f((x+1)
2
) = f(x
2
+2x+1) = f(x
2
) + 2f(x) + f(1) = xf(x) + 2f(x) + f(1).
T ta c phng trnh (x+1)(f(x)+f(1)) = xf(x) + 2f(x) + f(1), suy ra f(x) = f(1)x.
t f(1) = a, ta c f(x) = ax. Th li vo phng trnh ta thy nghim ng.
Vietnamese IMO Team Training Camp 2010
55 | Trn Nam Dng 6/2010


Vy f(x) = ax vi a R l tt c cc nghim ca bi ton.

Phng php to ra cc mi lin kt cng c th p dng hiu qu trong cc bi ton
phng trnh hm trn Q, N, Z. Ta xem xt mt s v d

Bi ton 6. Tm tt c cc hm s f : Q+ Q+ tho mn cc iu kin
i) f(x+1) = f(x) + 1 vi mi x thuc Q+;
ii) f(x
2
) = f
2
(x) vi mi x thuc Q+.
Gii. T iu kin i) ta d dng suy ra f(n) = n vi mi n thuc Z v f(r+n) = f(r) + n vi
mi r thuc Q v n thuc Z. By gi ta tnh f(r) vi
q
p
r = . tng ta s tnh f((r+q)
2
)
theo f(r) bng hai cch.
Trc ht f((r+q)
2
) = f
2
(r+q) = (f(r) + q)
2
(1)
Mt khc f((r+q)
2
) = f(r
2
+2p+q
2
) = f(r
2
) + 2p + q
2
= f
2
(r) + 2p + q
2
(2)
T (1) v (2) ta suy ra f
2
(r) + 2qf(r) + q
2
= f
2
(r) + 2p + q
2
=> f(r) = p/q = r.

Vy f(r) = r vi mi r thuc Q.

Bi ton 7. Tm tt c cc hm s f: N N sao cho
f(m
2
+n
2
) = f
2
(m) + f
2
(n) vi mi m, n thuc N
Gii. Cho m = n = 0, ta c f(0) = 2f
2
(0), suy ra f(0) = 0. Cho m = 1, n = 0, ta c f(1)
= 0 hoc f(1) = 1. Ta xt trng hp f(1) = 1, trng hp f(1) = 0 xt tng t. Vi f(1) =
1, ta ln lt tnh c
f(2) = f(1
2
+1
2
) = f
2
(1) + f
2
(1) = 2
f(4) = f(2
2
+0
2
) = f
2
(2) + f
2
(0) = 4
f(5) = f(2
2
+1
2
) = f
2
(2) + f
2
(1) = 5
Nhng lm sao tnh, chng hn f(3)? R rng f(3) khng th tnh c theo s trn
c, v 3 khng biu din c di dng tng ca hai bnh phng.

Ta nh li mt bi ton lp 3. C 1 ci cn a vi 2 qu cn 1kg, 5kg v 1 bao ng
nng 10kg. Hy cn ra 7kg ng bng 1 ln cn. R rng, vi cch cn thng thng
th ta ch cn c 1kg ng, 4kg ng (5-1), 5 kg ng v 6kg ng. Tuy nhin,
nu tinh 1 cht, ta c th c phng n cn c 7kg ng nh sau: t vo a bn
tri qu cn 1kg v 10kg ng, a bn phi l qu cn 5kg, sau chuyn dn ng
t bn tri sang bn phi sao cho cn cn bng, khi s ng cn li a bn phi
l 7kg !

By gi ta cng th thut tng vi bi ton ny. Ta khng tnh c trc tip f(3) nhng
ta li c f
2
(5) = f(25) = f(3
2
+4
2
) = f
2
(3) + f
2
(4). T ta c f(3) = 3.

Tng t nh vy ta c th tnh c f(6) nh vo ng thc 6
2
+ 8
2
= 10
2
, trong f(8)
= f(2
2
+2
2
) = 2f
2
(2) = 8, f(10) = f(3
2
+1
2
) = f
2
(3) + f
2
(1) = 10.
Vietnamese IMO Team Training Camp 2010
56 | Trn Nam Dng 6/2010

Tip tc, tnh f(7), ta 7
2
+ 1 = 50 = 5
2
+ 5
2
, t f(7) = 7. Cng nh th, do 11
2
+
2
2
= 10
2
+ 5
2
nn ta suy ra f(11) = 11.

Cch lm ny c th tng qut ho nh th no? tng l nu m
2
+ n
2
= p
2
+ q
2
(1) th
f
2
(m) + f
2
(n) = f
2
(p) + f
2
(q). Do nu ta tnh c f(n), f(p), f(q) th f(m) cng s tnh
c.

Lm th no c c nhng ng thc dng (1) dng tng qut, cho php ta chng
minh f(n) = n vi mi n bng quy np? Ch rng (1) c th vit li thnh (m-p)(m+p) =
(q-n)(q+n) = N. Do nu chn nhng s N c 2 cch phn tch thnh tch ca nhng s
c cng tnh chn l, ta s tm c nghim cho (1). Chn N = 8k = 2.4k = 4.2k v N =
16k = 4.4k = 8.2k, ta c h
m p = 2, m+p = 4k, q n = 4, q + n = 2k
v
m p = 4, m+p = 4k, q n = 8, q + n = 2k
T c cc hng ng thc tng ng
(2k+1)
2
+ (k-2)
2
= (2k-1)
2
+ (k+2)
2
v
(2k+2)
2
+ (k-4)
2
= (2k-2)
2
+ (k+4)
2

T hai ng thc ny, vi ch l ta chng minh c f(n) = n vi n = 0, 1, 2, 3, 4, 5,
6, ta d dng chng minh bng quy np c rng f(n) = n vi mi n N.

Trng hp f(1) = 0, cng bng cch l lun nu trn ta suy ra f(n) = 0 vi mi n thuc
N.

Bi tp.

1. Tm tt c cc hm s f: Q Q tho mn cc iu kin
i) f(x+1) = f(x) + 1 vi mi x thuc Q;
ii) f(x
3
) = f
3
(x) vi mi x thuc Q;

2. Tm tt c cc hm f: R \ {0} R tho mn ng thi cc iu kin
i) f(1) = 1;
ii)
|
|

\
|
+ |

\
|
=
|
|

\
|
+ y
f
x
f
y x
f
1 1 1

iii) (x+y)f(x+y) = xyf(x)f(y)
vi mi x, y m xy(x+y) 0.

3. Tm tt c cc hm s f: R R tho mn
f(x
5
y
5
) = x
2
f(x
3
) y
2
f(y
3
) vi mi x, y thuc R.

4. Tm tt c cc hm s f: Z Z tho mn iu kin
Vietnamese IMO Team Training Camp 2010
57 | Trn Nam Dng 6/2010

f(a
3
+b
3
+c
3
) = f
3
(a) + f
3
(b) + f
3
(c)
vi mi a, b, c thuc Z.

5. Cho hm s f: R R tho mn iu kin
i) f(x
2
) = f
2
(x) vi mi x thuc R;
ii) f(x+1) = f(x) + 1 vi mi x thuc R.
Chng minh rng f(x) = x.

3. Tnh gi tr ca hm s ti mt im c bit bng cch lp phng trnh

Trong phn trn, chng ta c th thy rng phng php lp phng trnh c th gip
chng ta tm c f(x) vi mi x, ni cch khc l gii c phng trnh hm. Trong
tnh hung tng qut, khng phi lc no ta cng thc hin c iu . Mi lin kt c
th ch tn ti i vi mt s cc gi tr c th. Ta xt mt v d:

Bi ton 8. Cho f l hm s khng gim xc nh trn on [0, 1], tho mn ng thi
cc iu kin
1) f(0) = 0
2) f(1x) = 1 f(x) x [0, 1];
3) f(x/3) = f(x)/2 x [0, 1].
Hy tnh f(1/13), f(1/7).

Gii. Tng t nh bi ton 4, ta thit lp th c nh l cc s thc thuc [0, 1] v
x c ni vi 1 x v x/3 (l cc i lng c th tnh c nu bit f(x)). Ta thit lp
chui

13
1
13
3
13
9
13
4
1
13
4
13
12
13
1
1
13
1
=
Nh vy, nu t y f =
|

\
|
13
1
th ta ln lt c
8
1
13
1
,
4
1
13
3
,
2
1
2
1
1
13
9
,
2
1
13
4
, 1
13
12 y
f
y
f
y y
f
y
f y f
+
= |

\
| +
= |

\
| +
=

= |

\
|
= |

\
|
= |

\
|

Nh vy ta phi c
8
1 y
y
+
= , t .
7
1
= y Vy .
7
1
13
1
=
|

\
|
f

Lu rng li gii ni trn l rt c th cho s 1/13. V d vi 1/7 th chui nh trn
s khng ng c

7
5
7
2
7
6
7
1

Nh vy, cch gii trn y khng p dng cho 1/7. tm gi tr f(1/7), ta cn n cc
iu kin f khng gim v f(0) = 0 (cc iu kin ny khng cn n khi tnh f(1/13)!).
C th, t cc iu kin bi, ta suy ra
f(0) = 0, f(1) = 1, f(1/3) = f(1)/2 = 1/2, f(2/3) = 1 f(1/3) = 1/2
Vietnamese IMO Team Training Camp 2010
58 | Trn Nam Dng 6/2010

T , do hm f khng gim nn ta suy ra f(x) = 1/2 vi mi x [1/3, 2/3]. p dng tnh
cht 3, ta suy ra f(x) = 1/4 vi mi x x [1/9, 2/9]. M 1/9 < 1/7 < 2/9 nn t y ta suy
ra f(1/7) = 1/4.

Trong cc bi ton phng trnh hm trn tp s nguyn, cc mi lin h dng bt ng
thc gia cc s cng c th l cha kho tm c gi tr ca hm s ti mt im.

Bi ton 9. Cho hm s f: N* N* tho mn ng thi cc iu kin sau
i) f(2) = 4 ;
ii) f(mn) = f(m).f(n) vi mi m, n thuc N* ;
iii) f(m) > f(n) vi mi m > n.
Hy tm f(3).

Gii. D thy hm s f(n) = n
2
tho mn c ba iu kin. Mt cch t nhin, ta ngh n
vic chng minh f(3) = 9.

Thng tin duy nht m ta c th c c t hai iu kin u l f(2
k
) = 4
k
. R rng, nu
khng c iu kin iii) th f(3) c th nhn gi tr tu , do f(3) khng tnh c.

Vi iu kin iii), ta c 4 = f(2) < f(3) < f(4) = 16, suy ra f(3) ch c th nhn gi tr trong
{5, 6, , 15}.


Tuy nhin, ch cn xt thm bt ng thc 8 < 9 l ta c th suy ra 64 = f(8) < f(9) =
f
2
(3), suy ra f(3) > 8. V bt ng thc 27 < 32 cho ta f
3
(3) = f(27) < f(32) = f
5
(2) =
1024, suy ra f(3) 10.

Nh vy, f(3) ch cn c th nhn 1 trong 2 gi tr 9 hoc 10.

Ta tm cch loi gi tr 10 bng 1 bt ng thc st hn. Tm kim trong cc lu tha ca
2 v 3, ta tm c cp bt ng thc 3
5
= 243 < 256 = 2
8
. Nu f(3) = 10 th t bt ng
thc trn ta s suy ra 10
5
< 4
8
100000 < 65536, mu thun.

Vy ch cn trng hp f(3) = 9. Kt hp vi nhn xt ban u l hm s f(n) = n
2
tho
mn tt c cc iu kin ca bi, ta kt lun f(3) = 9.

Ta c th s dng tng trn tnh f(n) vi n bt k, c th l chng minh f(n) = n
2

vi mi n. Ni chung, trong mt phng trnh hm tng qut th vic tm gi tr hm s
ti tng im nh vy c th s gp kh khn. Tuy nhin, c th th
y rng, trong mt s phng trnh hm th vic tm cc gi tr c bit nh f(0), f(1), f(-1)
ng vai tr then cht trong li gii ca bi ton. Di y ta xem xt mt s v d.

Bi ton 10. Tm tt c cc hm s f: R R tho mn iu kin
Vietnamese IMO Team Training Camp 2010
59 | Trn Nam Dng 6/2010

f(xy) + f(x+y) = f(x)f(y) + f(x) + f(y) (1)

Gii. Phng trnh trn l tng ca hai phng trnh
(2) f(x+y) = f(x) + f(y)
(3) f(xy) = f(x)f(y)
Ch l theo l thuyt v phng trnh hm Cauchy th nu hm s f: R R tho mn
ng thi hai phng trnh (2) v (3) th f(x) = x hoc f(x) 0. Do vy, ta ch cn chng
minh l t (1) suy ra (2) v (3) l xong.

tch c cc phng trnh trong (1), ta quan tm n tnh chn, l ca f. Chng hn
nu f l hm s l th thay y bng y, ta s thu c
f(-xy) + f(x-y) = f(x)f(-y) + f(x) + f(-y) (4)
Cng (1) v (4), ch f l hm l, ta s thu c
f(x+y) + f(x-y) = 2f(x)
T phng trnh ny v f(0) = 0 (suy ra t (1) bng cch cho x = y = 0), ta d dng suy ra
f(x+y) = f(x) + f(y) vi mi x, y, tc l (1) tch c.

Quay tr li vi bi ton. Nh vy, ta quan tm n tnh chn l ca hm s f.

Mt cch t nhin, ta thay y = -1 trong (1) th c
f(-x) + f(x-1) = f(x)f(-1) + f(x) + f(-1) (5)
Nu tm c f(-1) th c th kho st c tnh chn l ca f. Ta tm cch tnh f(-1).

Ch rng trong phng trnh (5) c n 3 n s l f(-x), f(x-1), f(x). iu ny khng
cho php chng ta thit lp dy truy hi tnh cc gi tr f. Mt la chn khc l thay y
= 1 vo (1) c
f(x) + f(x+1) = f(x)f(1) + f(x) + f(1)
Suy ra
f(x+1) = f(1)f(x) + f(1) (6)

Nh vy, theo ngn ng th dng trong cc bi ton 4, 8, ta tm c mi lin
kt x x+1.

T y, nu t f(1) = a th ta ln lt tnh c f(2) = a
2
+ a, f(3) = a
3
+ a
2
+ a, f(4) = a
4

+ a
3
+ a
2
+ a.
Nu tip tc dy ny th s tnh c f(n) theo f(1). Tuy nhin, iu ta cn tm l f(1). V
th, ta cn tm mt phng trnh khc vi cng thc truy hi ni trn. iu ny c th c
c ngay nu ta thay x = y = 2 vo (1). Khi
f(4) + f(4) = f
2
(2) + 2f(2)
Thay cc gi tr ca f(4) v f(2) tnh theo a trn vo, ta c phng trnh
2(a
4
+a
3
+a
2
+a) = (a
2
+a)
2
+ 2(a
2
+a)
a
4
= a
2

T suy ra a ch c th nhn cc gi tr a = 0, a = -1, a = 1.
Vietnamese IMO Team Training Camp 2010
60 | Trn Nam Dng 6/2010

+ Nu a = 0 th t (6) ta suy ra f(x) = 0 vi mi x. y l mt nghim ca phng trnh
hm.
+ Nu a = 1 th t (6) ta suy ra f(x+1) = f(x) 1, tc l f(x) + f(x+1) = 1 vi mi x.
T y, thay x = 1, ch l f(0) = 0, ta c f(1) = 1. Thay vo (5), ta c f(x) +
f(x1) = 1. M f(x1) + f(x) = 1 nn t y suy ra f(x) = f(x), tc l f l hm chn.
By gi trong (1), ta thay y bng y th c
f(-xy) + f(x-y) = f(x)f(-y) + f(x) + f(-y) (4)
Tr (1) cho (4), ch l f l hm chn, ta suy ra f(x+y) f(xy) = 0 vi mi x, y. T y
suy ra f l hm hng. iu ny mu thun v f(0) = 0, f(1) = 1. Vy trng hp ny
khng xy ra.

+ Nu a = 1 th t (6) ta suy ra f(x+1) = f(x) + 1 (7). Thay x = 1, ta c f(1) = 1.
Thay vo (5), ta c f(x) + f(x1) = 1. Li c t (7) f(x1) = f(x) 1 nn t y ta suy
ra f(x) = f(x). Nh vy f l hm l. L lun nh phn tch ban u, ta c f(x) tho
mn h phng trnh (2), (3). p dng l thuyt phng trnh hm Cauchy, ta tm c
f(x) = x.

Bi ton 11. Tm tt c cc hm s f: R R tho mn iu kin
f(x
2
+ f(y)) = f
2
(x) + y (1)

Gii. Vn mu cht trong bi ton ny l chng minh c f(0) = 0. Tht vy, nu ta
chng minh c f(0) = 0 th thay y = 0 vo phng trnh hm, ta c
f(x
2
) = f
2
(x) (2)
Thay x = 0 vo (1), ta c
f(f(y)) = y (3)
Thay y bng f(y) trong (1), ch rng f(f(y)) = y v f
2
(x) = f(x
2
), ta c f(x
2
+y) =
f(x
2
) + f(y) vi mi x, y thuc R
T y d dng chng minh c rng
f(x+y) = f(x) + f(y) (4)
vi mi x, y thuc R.
Nh vy f tho mn phng trnh hm Cauchy. Ngoi ra, do (2) nn ta c f(x) 0 vi
mi x 0. Kt hp vi (4), ta suy ra f l hm s tng. Nh vy, theo l thuyt c bn v
phng trnh hm Cauchy, f(x) = ax vi mi x, trong a 0. Thay vo phng trnh
hm, ta c a(x
2
+ay) = a
2
x + y vi mi x, y, suy ra a = 1. Vy f(x) = x l nghim duy
nht ca phng trnh hm.

Tt cc l lun trn y u da trn gi thit l ta chng minh c f(0) = 0. By gi
ta s chng minh iu ny.

t f(0) = a. Thay y = 0 vo (1), ta c
f(x
2
+a) = f
2
(x) (5)
Thay x = 0 vo (5), ta c f(a) = a
2

Thay x = 0 vo (1), ta c
Vietnamese IMO Team Training Camp 2010
61 | Trn Nam Dng 6/2010

f(f(y)) = a
2
+ y (6)
By gi ta s tnh f(a
2
+f
2
(1)) bng hai cch. Mt mt
f(a
2
+f
2
(1)) = f(f
2
(1) + f(a)) = (f(f(1)))
2
+ a = (a
2
+1)
2
+ a
Mt khc
f(a
2
+f
2
(1)) = f(a
2
+f(1+a)) = (f(a))
2
+ 1 + a = a
4
+ a + 1
T y ta suy ra a
4
+ 2a
2
+ 1 + a = a
4
+ a + 1 suy ra a = 0. Tc l f(0) = 0. Li gii hon
tt.

Bi tp

1. Cho f l hm s khng gim xc nh trn on [0, 1], tho mn ng thi cc iu
kin
i) f(0) = 0
ii) f(1x) = 1 f(x) x [0, 1];
iii) f(x/3) = f(x)/2 x [0, 1].
a) Hy tnh f(18/1991);
b) Hy tnh f(1/n) vi n {1, 2, , 20}
c)* Chng minh rng tn ti duy nht mt hm s f tho mn ng thi cc iu kin
nu trn.

2. Tm tt c cc hm s f: N* N* tho mn cc iu kin
i) f(2) = 2;
ii) f(mn) = f(m).f(n) vi mi (m, n) = 1;
iii) f(m) > f(n) vi mi m > n.

3. Tm tt c cc gi tr k thuc N* sao cho tn ti hm s f: N* N* tho mn cc iu
kin
i) f(2) = k;
ii) f(mn) = f(m).f(n) vi mi m, n;
iii) f(m) > f(n) vi mi m > n.

4. Tm tt c cc hm s f: R R tho mn iu kin
f(xy) + f(x) + f(y) = f(x+y) + f(x)f(y) vi mi x, y thuc R.

5. Tm tt c cc hm s f : R R tho mn iu kin
(f(x+y))
2
= f(x)f(x+2y) + yf(y) vi mi x, y thuc R.

6. Tm tt c cc hm s f: R R tho mn iu kin
f(x
2
+ y + f(y)) = f
2
(x) + 2y
vi mi x, y thuc R.



Vietnamese IMO Team Training Camp 2010
62 | Trn Nam Dng 6/2010

Ti liu tham kho

[1] Nguyn Vn Mu, Phng trnh hm, Nh xut bn Gio dc 2001

[2] Nguyn Trng Tun, Bi ton hm s qua cc k thi Olympic, Nh xut bn Gio dc
2005.

[3] Phan c Chnh, L nh Thnh, Phm Tn Dng, Tuyn tp cc bi ton s cp,
Tp 1, i s, NXB i hc v THCN 1977

[4] Phan Huy Khi, Cc bi ton v hm s, Nh xut bn Gio dc 2007

[5] B.J.Venkatachala, Functional Equations A Problem Solving Approach, PRISM
2002.

[6] Pierre Bornsztein, Mobinool Omarjee, Cours Equations fonctionelles, Electronic
Edition 2003

[7] Titu Andreescu, Iurie Boreico, Functional Equations, Electronic Edition 2007
























Vietnamese IMO Team Training Camp 2010
63 | Trn Nam Dng 6/2010

Ti u t hp I: Cc bi ton ti u v h cc tp hp
Gil Kalai
Bi vit ny gii thiu v cc bi ton ti u lin quan n h cc tp hp. y l mt
trong nhng bi ging ca Gil Kalai ti seminar Cc khi nim c bn ti H TH
Hungari do David Kazhdan khi xng.

Paul Erds
1. Ba bi ton m u
Chng ta s bt u bng ba bi ton kh ging nhau, t rt d n rt kh.
Bi ton I: Cho N = {1,2, , n }. Hi kch thc ln nht ca h cc tp con ca
tho mn iu kin hai tp hp bt k thuc c phn giao khc rng? (Mt h nh vy
c gi l h giao nhau)
Tr li I: Kch thc ln nht l . Chng ta c th t c iu ny bng cc chn
tt c cc tp con cha phn t 1. Chng ta khng th t c con s ln hn, bi t
mi cp gm mt tp hp v phn b ca n, ta ch c th chn mt tp hp vo h cc
tp hp ca chng ta.
Bi ton II: Tm kch thc ln nht ca h cc tp con ca sao cho hai tp hp
bt k thuc c hp khc ?.
Vietnamese IMO Team Training Camp 2010
64 | Trn Nam Dng 6/2010

Cu tr li l hin nhin v bi ton II chng qua l bi ton I. Ch cn chuyn qua phn
b. V ta s c p s y nh bi ton I. Ta pht biu bi ton II khc.
Bi ton II mi: Tm kch thc ln nht ca h cc tp con ca sao cho vi hai
tp hp bt k thuc ta c giao ca chng khc rng v hp ca chng khc .
Mt v d ca mt h nh vy l tp hp tt c cc tp hp cha phn t 1 nhng khng
cha phn t 2. H ny c tp hp. Phi mt vi nm sau khi Erds xut bi
ton ny, Kleitman mi chng minh c l khng c h no ln hn vi tnh cht ny.
Bi ton III (Gi thuyt Erds-Sos) Cho l h cc th vi N l tp cc nh. Gi sa
rng hai th bt k ca h c chung mt tam gic. Hi c kch thc ln nht bng
bao nhiu?
Tng s cc th vi n nh l . (Ch : ta m cc th m khng tnh n s
ng cu ca chng.) Mt v d n gin ca h tp hp vi tnh cht cho l tt c cc
tp hp cha mt tam gic c nh no . V d tt c cc th cha cc cnh
{1,2},{1,3},{2,3}. H ny cha 1/8 ca tt c cc th. Tn ti hay khng mt h ln
hn cc th vi tnh cht cho? Erds v Sos a ra gi thuyt l cu tr li l khng
chng ta khng th tm c h ln hn. Gi thuyt ny hin nay vn l mt vn
m.

Vera Sos
2. Hai nh l c bn v h cc tp con vi cc iu kin giao cho trc.
nh l Erds-Ko-Rado: Mt h giao nhau cc k-tp con ca , trong cha
ti a tp hp.
Vietnamese IMO Team Training Camp 2010
65 | Trn Nam Dng 6/2010

nh l Fisher; deBruijn-Erds: H cc tp con ca sao cho hai tp hp khc nhau
bt k ca h c ng mt phn t chung c nhiu nht n phn t.
(Erds v deBruijn kt lun rng n im khng thng hng trn mt phng xc nh t
nht n ng thng. Hy th suy ra iu ny t nh l trn!)
Tt c cc k-tp con ca N cha phn t 1 cho v d v du bng cho nh l Erdos-Ko-
Rado. Vi nh l Erds deBruijn ta ly h {{1} , {1,2} {1,3} {1,n}} hoc thay tp
hp u tin bi {2,3,,n}, hoc ly mt mt phng x nh hu hn.

Mt phng Fano cc im x nh hu hn bc 2.
3. Php chng minh nh l deBruijn Erds bng i s tuyn tnh.
Php chng minh nh l Fisher; de Bruijn-Erds c th trnh by nh sau: Gi s rng
c m tp hp trong h. Xt ma trn lin kt ca h: Phn t (i,j) ca ma
trn ny bng 1 nu i thuc .
S kin ct yu y l cc ct ca ma trn lin kt l c lp tuyn tnh, t
y suy ra .
Lm th no c th chng minh rng cc ct l c lp tuyn tnh? Trc ht, chng ta
gi s rng c t nht 2 tp hp. Khi ta vit , v tnh tch trong
.
Ta lu rng nu i v j khc nhau th v .
Nh vy .
Vietnamese IMO Team Training Camp 2010
66 | Trn Nam Dng 6/2010

Nh vy tch ny ch c th bng 0 khi tt c cc h s u bng 0.
Chng minh ny l mt v d ca cc l lun v chiu trong t hp.
4. nh l Sperner
nh l Sperner (1927) khng nh rng kch thc ln nht ca h cc tp con ca N
l mt i xch i vi quan h bao hm l h s nh thc . Lubell tm c mt php
chng minh n gin v p cho nh l Sperner:
Gi l mt i xch nh vy v gi s rng n c tp hp k phn t. Ta tnh cc cp
trong = l mt hon v ca {1,2, ,n} v S l tp hp
thuc h c khi u l , c th l S={ } vi k no . Vi mi hon
v ta tm c nhiu nht mt khi u S trong h (do iu kin i xch). Nu S l
mt tp hp k phn t, ta c th tm c ng k! (n-k)! hon v vi S l khi u. Kt
hp hai s kin ny, ta c hay ni cch khc .
Bt ng thc ny (c gi l bt ng thc LYM) suy ra kt qu cn chng minh.

Bella Bollobas, mt trong nhng ngi pht hin ra bt ng thc LYM.
nh l Erds-Ko-Rado c mt php chng minh tng t vi chng minh trn.
tng l tnh cch cp trong l mt tp hp trong h, l hon v vng
quanh v l mt khong lin tc i vi .
Mt mt ta c (n-1)! hon v vng quanh v d dng thy rng vi mi mt hon v nh
vy, ta c th chn c nhiu nht k khong i mt giao nhau. Mt khc, vi mi tp
hp S c k!(n-k)! hon v vng quanh m trong S l mt khong lin tc.
Nh vy k! (n-k)! v iu ny cho chng ta nh l Erds Ko Rado.
Quay tr li mt cht v cu d dng nhn thy. Phn ny s dng iu kin .
Mt cch l lun cho phn ny nh sau: xt khong J m phn t tn cng bn tri l nm
bn tri nht v ch rng c k khong giao vi J m phn t tn cng bn tri nm
Vietnamese IMO Team Training Camp 2010
67 | Trn Nam Dng 6/2010

bn phi z. Mt cch khc l xt mt khong J bo c di k v ch rng 2k-2
khong c giao vi khong ny c chia thnh k-1 cp m mi cp cha hai khong
khng giao nhau.
5. nh l Turan v bi ton Turan
Trng hp c bit ca nh l Turan cho th khng cha tam gic c chng minh
bi Mantel vo nm 1907.
S ln nht cc cnh (k hiu l ) ca mt th n nh v khng ch tam gic t
c th hai phe y n nh vi kch thc hai phe cng gn nhau cng tt (c th
l [n/2] v [(n+1)/2].
nh l Turan dng tng qut c chng minh vo nhng nm 40 ca th k trc.
S ln nht cc cnh (k hiu l ) ca th n nh khng cha th con y
(r+1) nh t c ti th r phe y vi n nh, trong kch thc ca cc phn
cng gn nhau cng tt.

Paul Turan
Chng minh nh l Turan: Thc s nh l Turan khng kh; gn nh cch tip cn
no cng c th thnh cng. Sau y l mt cch tip cn nh vy: n gin, ta xt
trng hp tam gic. Xt nh v vi bc ln nht v chia cc nh cn li ca th thnh
hai phn: A cc nh k vi v, B l cc nh cn li. By gi ch l cc nh thuc
A lp thnh mt tp hp c lp (tc l khng c cnh ni gia cc nh ca A). Vi mi
nh thuc B ta xo i tt c cc cnh cha nh ny v thay vo , ni nh ny vi tt
c cc nh thuc A. rng trong th mi, bc ca mi nh u khng nh hn
bc th ban u. V, hn na, th mi l th hai phe (trong A l mt phe).
Cui cng ta ch cn chng minh l vi th hai phe th s cnh l ln nht khi hai phn
c s nh cng gn nhau cng tt.
Sau y l mt chng minh khc. Xo i mt nh ca th G vi n+1 nh khng ch
. S cnh ca th cn li khng vt qu . Thc hin iu ny i vi tt c cc
nh v ch rng mt mt cnh c tnh n-1 ln. Ta thu c rng s cnh trong G (v
Vietnamese IMO Team Training Camp 2010
68 | Trn Nam Dng 6/2010

ngha l ) khng vt qu phn nguyn trn ca . nh gi
ny cho chng ta kt qu chnh xc ca bi ton.
Chng ta kt thc chuyn tham quan th v ny bng bi ton m Turan a ra vo nm
1940. Chng ta mun tm s phn t ln nht ca tp hp cc b ba lp t {1,2,,n}
khng cha mt t din, tc l khng cha bn b ban c dng
{a,b,c},{a,b,d),{a,c,d},{b,c,d}.
Nu c gi cha bit p s, hy th a ra d on ca mnh. Turan a ra mt gi
thuyt ca mnh v gi thuyt ny hin nay vn cn l mt vn m.
Trn Nam Dng dch v gii thiu (t web site ca Seminar: Ti u t hp I)
Mt s ghi ch thm ca dch gi:
i xch: Trong mt tp sp th t (partial order set), mt i xch l mt h cc phn t
i mt khng so snh c vi nhau.
LYM: Lubell, Yamamoto, Meshalkin l nhng ngi chng minh nh l ny mt
cch c lp. Bollobas l ngi th t cng tm ra kt qu ny mt cch c lp.






















Vietnamese IMO Team Training Camp 2010
69 | Trn Nam Dng 6/2010

V k thi chn i tuyn Vit Nam tham d IMO 2010

Trn Nam Dng

K thi chn i tuyn Vit Nam tham d IMO 2010 (VTST 2010) c t chc trong hai
ngy 17, 18/4 vi s tham gia ca 42 th sinh n t cc tnh thnh v cc trng H.
Thnh phn tham d TST nm nay c kh nhiu im c bit. Hai i c lc lng
hng hu nht tham d TST l Ngh An (6 em) v Nng (5 em). Cc n v ln u
tin c th sinh d TST l B Ra Vng Tu (2 em), HSP Tp HCM (1 em). Mt im
ni bt na l Ph Yn vi 3 hc sinh tham d TST. Bn cnh , c th nhn mnh s
vng mt ca mt s n v c truyn thng nh Hi Phng, Thanh Ha, H Vinh hay s
xung sc ca cc n v c s m khc nh Vnh Phc, PTNK.

Nh thng l, thi chn i tuyn nm nay c 6 bi. im c bit l nm nay ch c
1 bi hnh, 1 bi i s. Cc phn mn S hc v T hp c u i hn vi 2 bi.
Bi 1 v 4 c coi l cc bi d ca k thi. Bi 2 thuc loi trung bnh. Cc bi 5, 6
c nh gi l kh v bi 3 l rt kh. nh gi chung l nm nay khng kh bng
nm ngoi.

Cng tc chm thi c tin hnh ngay sau k thi v kt qu chn c 6 thnh
vin tham d IMO 2010.

Tri vi d on lc quan ca nhiu ngi, im thi nm nay khng cao hn nhiu so vi
nm ngoi, im cao nht i tuyn l 24 v im thp nht l 18. Nh vy, k thi nm
nay mt ln na li khng nh chn l: Nu lm 3 bi chc n th s lt vo i tuyn.
Kt qu ny tip tc cho thy im yu chung ca th sinh vn l hai mng S hc v T
hp, cng nh khu trnh by ca th sinh rt c vn . Sau k thi, kh nhiu th sinh
tuyn b mnh lm c 4 bi nhng kt qu thc t cho thy khng phi nh vy.

C mt s kin cho rng cc bi ton 5, 6 khng ph hp vi thi chn i tuyn v
em li li th cho nhng ai bit nh l Hall v nh l Lucas. Tuy nhin, theo kin
ca chng ti, mc k thi chn i tuyn, nu chng ta mun c mt i tuyn
mnh, sc tn cng cc bi ton 3, 6 ca IMO th cn phi nng tm kin thc cng
nh suy lun ca hc sinh n mc cc nh l ny. Ch l cc nh l ny rt s
cp v cha ng nhiu phng php t duy v l lun p v hay.

Cui cng, xinh chc mng cc th sinh vt qua k thi Vietnam TST va qua, chc
i tuyn Vit Nam c thnh tch xut sc ti IMO 2010!




Vietnamese IMO Team Training Camp 2010
70 | Trn Nam Dng 6/2010

CHN I TUYN VN D THI TON QUC T

Ngy 1, 17/4/2010
Thi gian lm bi: 240 pht.

BI 1. (6 im) Cho tam gic ABC khng vung ti A c trung tuyn AM. D l mt
im chy trn AM. Gi (O
1
), (O
2
) ln lt l cc ng trn i qua D v tip xc vi
BC ti B v C. CA ct (O
2
) ti Q. BA ct (O
1
) ti P.
a) Chng minh rng tip tuyn ti P ca (O
1
) v tip tuyn ti Q ca (O
2
) phi ct nhau.
Gi giao im ny l S.
b) Chng minh rng S lun chy trn mt ng c nh khi D chy trn AM.

BI 2. (6 im) Vi mi s n nguyn dng, xt tp sau

Tm tt c n sao cho khng tn ti a khc b thuc T
n
sao cho a-b chia ht cho 110.

BI 3. (8 im) Hnh ch nht kch thc 1*2 c gi l hnh ch nht n. Hnh ch
nht 2*3 b di 2 gc cho nhau (tc cn c 4 ) gi l hnh ch nht kp. Ngi ta
ghp kht cc hnh ch nht n v hnh ch nht kp c bng 2008*2010. Tm s b
nht cc hnh ch nht n c th dng lt c nh trn.

Ngy 2, 18/4/2010
Thi gian lm bi: 240 pht.

BI 4. (6 im) Cho a, b, c l cc s thc dng tha mn iu kin
.
Chng minh rng
.

BI 5. (7 im) C n mi nc c k i din (n > k > 1). Ngi ta chia n.k ngi ny
thnh n nhm mi nhm c k ngi sao cho khng c 2 ngi cng nhm n t 1 nc.
Chng minh rng c th chn ra n ngi n t cc nhm khc nhau v n t cc nc
khc nhau.

BI 6. (7 im) Gi S
n
l tng bnh phng cc h s trong khai trin ca (1+x)
n
. Chng
minh rng S
2n
+ 1 khng chia ht cho 3.




Vietnamese IMO Team Training Camp 2010
71 | Trn Nam Dng 6/2010

Li gii v Nhn xt

Bi 1.
Ta c MB
2
= MC
2
nn M thuc trc ng phng ca (O
1
) v (O
2
). Suy ra DM l trc
ng phng ca 2 ng trn. Do A thuc trc ng phng ca 2 ng trn.
Suy ra AP.AB = AQ.AC => t gic BCPQ ni tip.
Gi tip tuyn ca (O
1
) l Px th xPB = PBC = PQA, suy ra Px tip xc vi (APQ)
hay (APQ) tip xc vi (O
1
). Tng t suy ra (APQ) tip xc vi c (O
1
) v (O
2
).
Tam gic APQ ng dng vi ACB nn APQ khng vung. Suy ra tip tuyn ti P v Q
phi ct nhau ti S.
V SP
2
= SQ
2
nn S thuc trc ng phng ca (O
1
) v (O
2
), chnh l ng thng AM,
hay S thuc mt ng thng c nh.

Bi 2.
t s(k,h, n) = 11(k + h) + 10(n
k
+ n
h
). Do s(h,k) = s(k, h) nn ta lun gi s k h.
Ta thy nu n m th s(k,h, n) s(k,h,m) mod 110, do ta ch cn tm cc n 11.
Xt s(6,6,n) s(1,1,n) = 110 + 20n(n
5
-1). Do nu n(n
5
-1) chia ht cho 11 th n khng
tha mn iu kin.
T y ta loi cc gi tr n = 1, 3, 4, 5, 9, 11.
Xt s(8,2,n) s(6,4,n) = 10(n
8
-n
6
+n
2
-n
4
) = 10(n
6
-n
2
)(n
2
-1). Do nu n
2
1 chia ht cho
11 th n khng tha mn iu kin. Vy ta loi gi tr n = 10.
Ta chng minh vi n = 2, 6, 7, 8 th s(k,h,n) s(k,h,n) khng chia ht cho 110 vi mi
b {k, h} {k,h}.
Trc ht bng cch th trc tip, ta thy rng vi n = 2, 6, 7, 8 th n
k
n
h
mod 11 vi
mi k h. (*)
Tht vy, nu s(k,h,n) s(k,h,n) chia ht cho 110 th 11(k+h-k-h) + 10(n
k
+n
h
-n
k
-n
h
)
chia ht cho 110, suy ra
k+h k h 0 mod 10 (1)
v
n
k
+ n
h
n
k
n
h
0 mod 11 (2)
T (1) suy ra k k h h (mod 10). T y, theo nh l nh Fermat, ta c n
k-k
n
h-h

(mod 11)
Vit (2) li thnh n
k
(n
k-k
-1) n
h
(n
h-h
-1) (mod 11).
Theo l lun trn th n
k-k
-1 n
h-h
-1 mod 11. Theo (*) th n
k-k
-1 n
h-h
-1 0. Nh vy
ta c th chia hai v cho n
k-k
-1 c n
k
n
h
(mod 11). T (*) suy ra k = h. T (1) suy
ra k = h. Nh th {k,h} = {k, h}. Ta c iu phi chng minh.

Nhn xt. C th a ra c trng ngn gn cho cc s n tha mn iu kin l: n phi l
cn nguyn thy modulo 11. Bi ton c th tng qut ha bng cch thay 11 bng 1 s
nguyn t bt k (v d nhin 10 c thay bng p-1 v 110 c thay bng p(p-1).

y l mt bi ton kh p. tng n gin nhng khng tm thng.

Vietnamese IMO Team Training Camp 2010
72 | Trn Nam Dng 6/2010

Bi 3. Ta chng minh s hnh ch nht n nh nht cn s dng l 1006.
Trc ht ta ch ra cch ghp kht to thnh hnh ch nht 2008 x 2010 vi 1006 hnh ch
nht n (v cc hnh ch nht kp).











Khi 1 Khi 2 Khi 3 Khi 4
Trn hnh v m t cch ghp hnh ch nht 10 x 16. Hnh ch nht 2010x2008 c th
c to thnh t cu hnh trn nh sau:
+ Thm dng bng cch chn thm cc khi c dng




vo gia cc khi hp thnh t 4 ct lin tip. Mi ln thm c 2 dng.
+ Thm ct bng cch lp li cc khi 4 ct lin tip (ch tnh tun hon ca cc khi
ny: Khi 1 ~ Khi 3, Khi 2 ~ Khi 4 ).
+ Theo nh cch ghp trn th ta chia hnh ch nht 2010x2008 thnh 502 khi, mi khi
gm 4 ct. khi 1 v khi 502 ta cn dng 3 hnh ch nht n (tng ng vi cc khi
1 v 4 nh trong v d trn). Cc khi cn li ta dng 2 hnh ch nht n.

Nh vy tng cng trong cch ghp trn ta dng 500 x 2 + 2 x 3 = 1006 hnh ch nht
n.

Xoay hnh ch nht 2010x2008 li, ta c hnh ch nht 2008 x 2010.

By gi ta chng minh phi cn t nht 1006 hnh ch nht n ph hnh ch nht
2008 x 2010. Xt mt php ph hp l, gi x, y, z, t ln lt l s hnh ch nht 1 x 2, 2 x
1, 2 x 3, 3 x 2 s dng trong php ph ny. T trng cc hng l, t en cng hng chn.
in s i vo hng th i. Ta c

Nhn xt 1. 2(x+y) + 4(z+t) = 2008 x 2010. (*)

Vietnamese IMO Team Training Camp 2010
73 | Trn Nam Dng 6/2010

Nhn xt 2. Xt trn ton bng, hnh ch nht khuyt v hnh ch nht 2 x 1 c s trng
bng s en. Suy ra s hnh ch nht 1 x 2 cc hng trng = s hnh ch nht 1 x 2
cc hng en = x/2. Nh vy x chn.

Nhn xt 3. Vi mi hnh ch nht ca php ph, ta xt tng cc s trong en tng
cc s trong trng. Khi i vi hnh ch nht 3 x 2 i lng ny = 0, vi hnh ch
nht 2 x 3 i lng ny bng 2, vi hnh ch nht 2 x 1 i lng ny bng 1. Cui
cng, tng cc s trong hnh ch nht 1 x 2 l mt s chn thuc [2, 2.2008]. Do ta c
bt ng thc
Tng cc s en Tng cc s trng = 1004.2010 (x/2)(2.2008-2) + y +
2z.
1004.2010 2007x + y + 2z. (**)

i ch hng v ct, ta c
1005.2008 2009y + x + 2t (***)
Cng (**) vi (***) ri (*), ta c
1005.2008 2007x+2009y 2009(x+y)
Suy ra x + y 1005.2008/2009 > 1004. V x, y chn nn x + y 1006.

Nhn xt.
1. y l bi ton kh nht ca ngy th nht v cng l bi ton kh nht ca c k thi.
im mu cht ca li gii l tm ra cch ph ti u v chng minh tnh ti u ca n.
Vic tm ra cch ph cho hnh ch nht 4 x 2n vi 4 hnh ch nht n l khng kh
nhng rt d dn n ng nhn l vi hnh ch nht 4m x 2n (2m < n) ta c th nhn cch
ph trn thnh cch ph ti u s dng 4m hnh ch nht n. Trong thc t, cch ph
ti u s dng 1 hnh ch nht kp thay hai hnh ch nht n cho cc khi k nhau.

2. Sau khi tm ng cch ph ti u, ta cn chng minh tnh ti u ca n. Mt cch
tip cn truyn thng l t mu. Tuy nhin, mt vi php th v sai cho thy cch t mu
en trng hoc A, B, C, D n gin khng gii quyt c vn v ta phi s dng n
nhng mi lin h su sc hn gia cc v s ha cc mi lin h ny bng cch a
trng s vo.

3. C kin cho rng bi s 3 ging vi mt bi ton thi chn i tuyn Vit Nam nm
1993. Tuy nhin, nu xem xt k li gii th s ging nhau ch l hnh thc. Bi 3 nm
nay kh hn rt nhiu so vi bi 1 nm 1993.

(VTST 1993, Bi 1) Gi hnh ch nht kch thc 2 x 3 (hoc 3 x 2) b ct b mt hnh
vung 1 x 1 mt gc l hnh ch nht khuyt n. Gi hnh ch nht kch thc 2 x 3
(hoc 3 x 2) b ct b hnh vung 1 x 1 hai gc i din l hnh ch nht khuyt kp.
Ngi ta ghp mt s hnh vung 2 x 2, mt s hnh ch nht khuyt n v mt s hnh
ch nht khuyt kp vi nhau, sao cho khng c hai hnh no chm ln nhau, to
thnh mt hnh ch nht kch thc 1993 x 2000. Gi s l tng s cc hnh vung 2 x 2
Vietnamese IMO Team Training Camp 2010
74 | Trn Nam Dng 6/2010

v hnh ch nht khuyt kp cn dng trong mi cch ghp hnh ni trn. Tm gi tr ln
nht ca s.

Li gii bi ton trn kh n gin (khng phi ngu nhin bi ny c t v tr bi
s 1 ca k thi): T mu cc hng xen k en trng th s c 997 hng en v 996 hng
trng, suy ra s en nhiu hn s trng l 2000. Mi mt hnh vung 2 x 2 v mi
mt hnh ch nht khuyt kp lun c ng 2 trng, 2 en, cn mi mt hnh ch nht
khuyn n c s en tr s trng = 1 hoc -1. V vy nu gi x, y, z ln lt l s
hnh vung 2 x 2, s hnh ch nht khuyt kp, s hnh ch nht khuyt n th ta ln
lt c:
1) 4x + 4y + 5z = 1993 x 2000 (s )
2) 2000 z (s en s trng)
T y suy ra
4x + 4y 1988 x 2000
Suy ra s = x + y 994000. Php ph ti u c th xy dng d dng nh cu hnh c bn
sau:










Bi 4.
p dng bt ng thc AM-GM, ta c

suy ra

Cng bt ng thc ny vi hai bt ng thc tng t, ta suy ra

Hn na, ta li c

V vy,
Vietnamese IMO Team Training Camp 2010
75 | Trn Nam Dng 6/2010

(1)
By gi, s dng bt ng thc c bn ta suy ra

tc
(2)
Kt hp (1) v (2), ta c ngay kt qu cn chng minh. ng thc xy ra khi v ch khi

Nhn xt.
nh gi l kh c bn v t nhin, ng
thi cng l bc mu cht n gin ha bt ng thc.
Sau khi tm c nh gi , ta s dng iu kin
mt cch trc tip a bt ng thc cn chng
minh v dng thun nht

n y, ta c nhiu phng hng chng minh khc nhau, trong c cch trnh by
trong li gii.

Bi 5.
Cch 1. Ta nhn xt rng vi iu kin ca bi th vi mi h thuc {1, 2, , n} th tp
hp cc i biu h nhm bt k s n t t nht h nc. Tht vy, h nhm gm hk i
din, m mi nc ch c k i biu suy ra s nc c i din trong h nhm ni trn
khng th di h nc.
Ta gi tnh cht h nhm bt k c i biu ca t nht h nc l tnh cht (*). chng
minh bi ton, ta chng minh rng ta c th chuyn bt cc i din ra khi cc nhm sao
cho
1) Mi nhm ch cn li mt i din
2) iu kin (*) vn c tha mn
R rng nu khi ta chn cc i din cn li trong phng th chnh l n ngi tha
mn yu cu bi ton, v theo iu kin (*) vi h = n th cc i din ny s l i biu
n t n nc.
Ta chng minh khng nh ni trn bng cch s dng nguyn l cc hn. Trong cc
cch chuyn bt cc i din ra khi cc nhm sao cho iu kin (*) vn c tha mn,
chn cch chuyn c s i din c chuyn ln nht. Ta chng minh rng vi cch
chuyn ny, mi nhm ch cn li 1 i din.
Vietnamese IMO Team Training Camp 2010
76 | Trn Nam Dng 6/2010

Tht vy, gi s c 1 nhm no no , chng hn nhm 1 cha t nht 2 ngi x, y.
Nu ta b i ngi x khi phng ny th theo cc chn cch chuyn trn, iu kin (*)
s khng cn c tha mn. Tc l tn ti q nhm i
1
, i
2
, , i
q
sao cho trong cc nhm
1 ( b ngi x), i
1
, i
2
, , i
q
(1)
ch cha nhiu nht i din ca q nc.
Tng t, tn ti p nhm j
1
, j
2
, , j
p
sao cho trong cc nhm
1 ( b ngi y), j
1
, j
2
, , j
p
(2)
ch cha nhiu nht i din ca p nc.
Trong cc ch s i v j, gi s c r ch s trng nhau: i
1
= j
1
, , i
r
= j
r
cn cc ch s kia
khc nhau. Khi do cc nhm i
1
, i
2
, , i
r
cha i din ca t nht r nc nn c t nht
r nc trng nhau hai danh sch (1) v (2). Do , hai danh sch (1) v (2) gp li cha
nhiu nht i din ca q + p r nc (3).
Mt khc, hp hai danh sch cc phng ni trn li, ta c danh sch cc phng phn
bit
1, i
1
, i
2
, , i
r
, i
r+1
, , i
k
, j
r+1
, , j
p

(nhm 1 thiu x v nhm 1 thiu y hp li thnh nhm 1).
p dng iu kin (*) ta thy hp ca hai danh sch ny cha i din ca t nht 1 + q +
p r nc, mu thun vi (3).
Bi ton c gii quyt hon ton.
Cch 2. li gii t nhin, chng ti trnh by thm phn dn dt. Trong li gii c th
b i phn in nghing.
thun tin trnh by, ta pht biu bi ton di dng tp hp: Cho A
1
, A
2
, , A
n
l n
tp con k phn t ca X = {1, 2, ,n}, trong mi phn t xut hin ng k ln. Khi
tm c a
1
, a
2
, , a
n
i mt khc nhau sao cho a
i
thuc A
i
.

D thy hp ca p tp hp bt k cha t nht p phn t.

tng l ta s chn a
1
mt cch bt k t tp A
1
, a
2
l mt phn t khc a
1
chn t A
2
.
Ta c chn ngu nhin theo nguyn tc nh th cho n khi khng chn c na. Tc l
ta gp trng hp gp tp hp A
p+1
trong tt c cc phn t ca n c chn
trc . Gi s a
i
thuc A
i
, i = 1, 2, , p l cc phn t c chn.

Ta t J
1
= { i {1, 2, , k}| a
i
A
k+1
}. Nu th ta
chn a thuc B. Gi s a thuc A
i1
th theo nh ngha, a
i1
thuc A
k+1
, khi ta i li,
chn a t A
i1
cn a
i1
t A
k+1
. Nh vy ta m rng cch chn cc phn t phn bit
n A
k+1
.

Trong trng hp B
1
= , ta li t J
2
= {i {1, 2, , k} | }. Nu
th ta chn a thuc B
2
. Gi s a thuc A
i1
th a
i1
s thuc
A
i2
vi i
2
no thuc J
1
, suy ra do a
i2
thuc A
k+1
. By gi ta chn a t A
i1
, a
i1
t A
i2

v a
i2
t A
k+1
. Nh vy ta m rng cch chn cc phn t phn bit n A
k+1
.

Vietnamese IMO Team Training Camp 2010
77 | Trn Nam Dng 6/2010

Nu B
2
= th ta li tip tc thc hin qu trnh trn .

By gi vo li gii chnh.
Ta xt th c hng G gm p+1 nh: 1, 2, , p, p+1. nh i c ni n nh j nu
a
i
thuc A
j
. Ta t
J = { i {1, 2,, p+1}| c ng i t i n p+1}

Vi mi i {1, 2,, p}, ta c a
i
A a
i
thuc A
j
vi j no thuc J c cnh ni
t i n j J G cha ng i t i n p+1 i J.

T {i | i {1, 2,, p}, a
i
A} = J \ {p+1} = J*.
t B = A \ {a
i
| i J*}. Khi B {a
1
, a
2
,, a
p
} = v |B| = |A| - |J*| = |A| - |J| + 1
1.

Xt a B. Khi a A
j
vi j J no . R rng j p+1. Nh vy c 1 ng i t j
n p+1. Gi s l j
0
=j, j
1
,, j
l
= p+1. Ch rng I = {j
0
, j
1
,, j
l
} J. By gi nu
t b
j0
, b
j1
, , b
jl
tng ng l a, a
j0
, , a
j(l-1)
th r rng b
i
A
i
vi mi i thuc I. t
b
p+1
= a
jl
, b
i
= a
i
vi mi i I. Khi b
1
, b
2
, , b
k+1
l cc phn t phn bit ly t A
1
,
A
2
, , A
p+1
tng ng.

Nh vy ta m rng c cho p+1 tp hp. Bi ton c chng minh hon ton.

Cch 3. Li gii ny s dng nh l Hall sau y:
B : Cho A
1
, A
2
, , A
n
l mt h cc tp con ca mt tp hp X tha mn iu kin
| vi mi I X. (*)
Khi tn ti a
1
, a
2
, , a
n
phn bit sao ca X sao cho a
i
A
i
vi mi i = 1, 2, , n.
Chng minh.
Ta chng minh bng quy np theo n. Vi n = 1, kt lut ca b l hin nhin.

Gi s b ng vi mi h tp con F c |F| < n. Xt h F = (A
1
, A
2
, , A
n
). Ta xt hai
trng hp.
Trng hp 1: Tn ti mt tp ch s I {1, 2, , n}, | I | < n sao cho | .
t J = {1, 2, , n} \ I, , A
j
* = A
j
\ A vi mi j J. Ta chng minh h cc
A
j
* tha mn iu kin (*). T p dng gi thit quy np cho cc h A
i
, i I, A
j
*, j
J ta suy ra iu phi chng minh.

Gi s ngc li, tn ti K J sao cho |
Khi , do nn ta c
| |
Mt khc, do nn v tri ca ng thc trn bng
| Mu thun.
Vietnamese IMO Team Training Camp 2010
78 | Trn Nam Dng 6/2010


Trng hp 2: Vi mi I {1, 2, , n}, | I | < n, ta c | . Khi chn mt
phn t bt k a
1
t A
1
. t A
i
* = A
i
\{a
1
} vi mi i = 2, 3, , n. Khi vi mi I { 2,
, n}, ta c
|
Nh vy h (A
2
*, , A
n
*) tha mn iu kin (*) v theo gi thit quy np, ta c th
chn c cc phn t phn bit a
2
, a
3
, , a
n
sao cho a
i
A
i
* vi mi i = 2, , n. Kt
hp vi a
1
ta c cc phn t phn bit a
1
, a
2
, , a
n
trong a
i
A
i
.

Quay tr li bi ton, bng cch m hnh ha cc i din ca cc nc nh cc phn t,
cc nhm nh cc tp con, bng cch l lun tng t nh li gii 1, ta thy cc tp con
ny tha mn iu kin (*) v do tm c cch chn tha mn yu cu bi ton.

Nhn xt. Vi k=1 v k = n, kt lun ca bi ton l hin nhin. Vi k = 2, ta c mt
cch gii kh n gin nh sau: nhm 1, ta ly ra 1 i din bt k thuc nc i
1
, tip
theo, ta n nhm c ngi cn li ca nc i
1
, ly ngi cn li trong nhm (thuc
nc i
2
) lm i din cho nhm ny, li chuyn sang nhm c ngi cn li ca nc i
2

Nu qu trnh ny c th ko di mi n ht n nhm th xong. Nu khng s xy ra
trng hp sau khi chn i din cho nhm k thuc nc i
k
th ngi cng nc vi cn
li trong nhm ny li thuc nhm 1. Nh th cc nhm 1, 2, , k to thnh mt xch, ta
loi xc ny i v lm vic vi nhng nhm cn li bng cch tng t.

Bi 6.
Ta c .
So snh h s ca 2 bn ta c
.
Tip theo ta chng minh khng chia ht cho .
Gi s 2n c biu din tam phn l

vi a
i
{0, 1, 2}.
Trng hp 1: cc a
i
thuc tp {0, 1}, Khi a
0
+ a
1
+ + a
k
= 2p.
4n c biu din tam phn l (do a
i
= 0 hoc 1) v theo nh l Lucas, ta
c

Trng hp 2: tn ti i nh nht m a
i
= 2, khi h s tng ng ca 4n l 1. Do
Vietnamese IMO Team Training Camp 2010
79 | Trn Nam Dng 6/2010


nn


Nhn xt.
V nh l Lucas khng c s dng nn th sinh khi s dng phi chng minh
li nh l ny.
Cng c th trnh by li gii trc tip khng thng qua nh l Lucas, tuy nhin
c l khng th trnh khi vic s dng h tam phn v l lun t hp di y.
Cch tip cn ca bi, yu cu chng minh S
2n
+ 1 khng chia ht cho 3 c hai
mc ch: 1) Kim tra xem th sinh c tnh c S
2n
khng? V y cng l ch
cho im.
2) a ra bn cht t hp ca t gi n cch khai trin a thc
theo m-un.
* hon chnh li gii, chng ti trnh by cch chng minh nh l Lucas.
nh l Lucas. Cho m v n l cc s nguyn khng m, p l s nguyn t v
m = m
k
p
k
+ m
k-1
p
k-1
+ + m
1
p + m
0

n = n
k
p
k
+ n
k-1
p
k-1
+ + n
1
p + n
0

l cc biu din p phn ca m v n tng ng. Khi
.
Chng minh. Ta lm vic trn cc a thc vi h s c xt theo modulo p. Do
vi mi k = 1, 2, , p-1 nn ta c
(1+ x)
p
1 + x
p
(mod p)
T bng quy np ta suy ra
(mod p)
Nh vy ta c
(mod p)
H s ca x
n
v tri l . Do biu din p phn ca n l duy nht nn h s ca x
n

v phi l T y ta c iu phi chng minh.

Li gii trn y c s dng p n chnh thc, li gii v tng ca cc bn LTL,
Phm Minh Khoa, V Quc B Cn, Traum v mt s tho lun khc trn din n
www.mathscope.org.
Vietnamese IMO Team Training Camp 2010
80 | Trn Nam Dng 6/2010

Bt ng thc: Mt s v d v bi tp chn lc

1. (USA MO 2004) Cho a, b, c l cc s thc dng. Chng minh rng
(a
5
a
2
+ 3)(b
5
b
2
+ 3)(c
5
c
2
+ 3) (a+b+c)
3


2. (IMO 2005) Chng minh rng nu a, b, c l cc s dng c tch ln hn hay bng 1
th
0
2 2 5
2 5
2 2 5
2 5
2 2 5
2 5

+ +

+
+ +

+
+ +

b a c
c c
a c b
b b
c b a
a a


3. (Kvant) Cho a, b, c l cc s thc dng tho mn iu kin a + b + c = 1. Chng minh
rng ta lun c
25 ) ( 48
1 1 1
+ + + + + ca bc ab
c b a


4. (Mathlinks) Cho a, b, c, x, y, z l cc s thc tha mn iu kin
(a+b+c)(x+y+z) = 3, (a
2
+b
2
+c
2
)(x
2
+y
2
+z
2
) = 4
Chng minh rng ax + by + cz 0.

5. (Vit Nam 2002) Cho x, y, z l cc s thc tho mn iu kin x
2
+ y
2
+ z
2
= 9. Chng
minh rng 2(x+y+z) xyz 10.

6. Cho x, y, z l cc s thc tho mn iu kin x + y + z = 0 v x
2
+ y
2
+ z
2
= 6. Tm gi
tr ln nht ca biu thc F = x
2
y + y
2
z + z
2
x.

7. (Vasile Cirtoaje) Cho cc s thc khng m tha mn iu kin a + b + c = 4. Chng
minh rng
a
2
b + b
2
c + c
2
a + abc 4.

8. (IMO 1999) Cho n 2 l mt s nguyn dng c nh, tm hng s C nh nht sao
cho vi mi s thc khng m x
1
, x
2
, , x
n



< =
|

\
|
+
n j i
n
i
i j i j i
x C x x x x
1
4
1
2 2
) (

9. Cho a, b, c l cc s thc dng cho trc v x, y, z l cc s thc dng thay i
tho mn iu kin xyz = ax + by + cz. Chng minh rng gi tr nh nht ca x + y + z
bng d ab b a d ca a c d bc c b / 2 / 2 / 2 + + + + + + + + , trong d l s thc dng xc
nh bi phng trnh
1 =
+
+
+
+
+ d c
c
d b
b
d a
a
.

Vietnamese IMO Team Training Camp 2010
81 | Trn Nam Dng 6/2010

10. (Romanian TST 2007) Cho n 2 v x
1
, , x
n
; y
1
, , y
n
l 2n s thc tho mn iu
kin


= = =
= = =
n
i
n
i
i i i
n
i
i
b a b a
1 1
2
1
2
, 0 , 1 , 1
Chng minh rng .
2
1
2
1
n b a
n
i
i
n
i
i
|

\
|
+ |

\
|

= =


11. (IMO Shortlist 2007) Cho a
1
, a
2
, , a
100
l cc s thc khng m tho mn iu kin
a
1
2
+ a
2
2
+ + a
100
2
= 1. Chng minh rng
a
1
2
a
2
+ a
2
2
a
3
+ + a
100
2
a
1
< 12/25.

12. IMO Short List 2003) Cho n 2 l s nguyn dng v x
1
, x
2
, , x
n
, y
1
, y
2
, , y
n
l
2n s thc dng. Gi s z
2
, z
3
, , z
2n
l cc s thc dng sao cho z
2
i+j
x
i
y
j
vi mi i,
j thuc {1, 2,,n}. t M = max{z
2
, z
3
,, z
2n
}. Chng minh rng
|

\
| + +
|

\
| + +
|

\
| + + + +
n
y y
n
x x
n
z z z M
n n n
... ...
2
...
1 1 2 3 2


13. Cho a
1
, a
2
, , a
n
l cc s thc sao cho a
1
2
+ a
2
2
+ + a
n
2
= 1. Tm gi tr ln nht
ca biu thc a
1
a
2
+ a
2
a
3
+ + a
n-1
a
n
.

14. Cho x
1
, x
2
, , x
n
l cc s dng. Gi A l s nh nht trong cc s
,
1
,
1
,...,
1
,
1
,
1 2
3
1
2 1
n n
n
x x
x
x
x
x
x x

+ + + cn B l s ln nht trong cc s ny. Chng minh


rng gi tr ln nht ca A bng gi tr nh nht ca B v hy tm gi tr ny.

15. Tng n s thc dng x
1
, x
2
, x
3
, ..., x
n
bng 1. Gi S l s ln nht trong cc s
x
1
/(1 + x
1
), x
2
/(1 + x
1
+ x
2
), ..., x
n
/(1 + x
1
+ x
2
+ ... + x
n
). Tm gi tr nh nht ca S. Vi
nhng gi tr no ca x
1
, x
2
, x
3
, ..., x
n
th gi tr ny t c?

You might also like